Endocrine, Metabolic & Nephrology MCQS: Thyroid Eye Disease
Endocrine, Metabolic & Nephrology MCQS: Thyroid Eye Disease
Q-1 • inability to close the eye lids may lead to sore, dry eyes. If
A 54-year-old female presents to the Emergency Department severe and untreated patients can be at risk of exposure
concerned about double vision. She is noted to have keratopathy
exophthalmos and conjunctival oedema on examination and
a diagnosis of thyroid eye disease is suspected. What can be Management
said regarding her thyroid status? • topical lubricants may be needed to help prevent corneal
inflammation caused by exposure
A. Hyper- or euthyroid • steroids
B. Hypothyroid • radiotherapy
C. Hyperthyroid • surgery
D. Hypo- or euthyroid
E. Eu-, hypo- or hyperthyroid Monitoring patients with established thyroid eye disease
ANSWER: For patients with established thyroid eye disease the following
Eu-, hypo- or hyperthyroid symptoms/signs should indicate the need for urgent review by
an ophthalmologist (see EUGOGO guidelines):
EXPLANATION: • unexplained deterioration in vision
Whilst thyroid eye disease is mostly associated with • awareness of change in intensity or quality of colour
hyperthyroidism secondary to Graves' disease there is not vision in one or both eyes
always an association. A minority of patients will either be • history of eye suddenly 'popping out' (globe subluxation)
euthyroid or hypothyroid. • obvious corneal opacity
• cornea still visible when the eyelids are closed
It is also known that the severity of thyroid eye disease is not
• disc swelling
related to the degree of thyrotoxicosis in Graves' disease.
Q-2
THYROID EYE DISEASE
Which one of the following statements regarding impaired
Thyroid eye disease affects between 25-50% of patients with
glucose regulation is correct?
Graves' disease.
A. All patient should have a repeat oral glucose tolerance
Pathophysiology
test every 2 years
• it is thought to be caused by an autoimmune response
B. Patients with impaired glucose tolerance are more likely
against an autoantigen, possibly the TSH receptor →
to develop diabetes than patients with impaired fasting
retro-orbital inflammation
glycaemia
• the inflammation results in glycosaminoglycan and C. Impaired glucose tolerance (IGT) is defined as a fasting
collagen deposition in the muscles glucose greater than or equal to 6.1 but less than 7.0
mmol/l
Prevention D. Around 1 in 20 adults in the UK have impaired glucose
• smoking is the most important modifiable risk factor for regulation
the development of thyroid eye disease E. Patients should be offered pioglitazone if lifestyle
• radioiodine treatment may increase the inflammatory changes fail to improve their glucose profile
symptoms seen in thyroid eye disease. In a recent study
of patients with Graves' disease around 15% developed, ANSWER:
or had worsening of, eye disease. Prednisolone may help Patients with impaired glucose tolerance are more likely to
reduce the risk develop diabetes than patients with impaired fasting
glycaemia
Features
• the patient may be eu-, hypo- or hyperthyroid at the time EXPLANATION:
of presentation PREDIABETES AND IMPAIRED GLUCOSE REGULATION
• exophthalmos Prediabetes is a term which is increasingly used where there is
• conjunctival oedema impaired glucose levels which are above the normal range but
• optic disc swelling not high enough for a diagnosis of diabetes mellitus. The term
• ophthalmoplegia includes patients who have been labelled as having either
impaired fasting glucose (IFG) or impaired glucose tolerance
(IGT). Diabetes UK estimate that around 1 in 7 adults in the UK • impaired glucose tolerance (IGT) is defined as fasting
have prediabetes. Many individuals with prediabetes will plasma glucose less than 7.0 mmol/l and OGTT 2-hour
progress on to developing type 2 diabetes mellitus (T2DM) value greater than or equal to 7.8 mmol/l but less than
and they are therefore at greater risk of microvascular and 11.1 mmol/l
macrovascular complications. • people with IFG should then be offered an oral glucose
tolerance test to rule out a diagnosis of diabetes. A result
Terminology below 11.1 mmol/l but above 7.8 mmol/l indicates that
• Diabetes UK currently recommend using the term the person doesn't have diabetes but does have IGT
prediabetes when talking to patients and impaired
glucose regulation when talking to other healthcare Q-3
professionals A 29-year-old female who is 14 weeks into her first
• research has shown that the term 'prediabetes' has the pregnancy is investigated for excessive sweating and tremor.
most impact and is most easily understood Blood tests reveal the following:
ANSWER:
Propylthiouracil
EXPLANATION:
Propylthiouracil is traditionally taught as the antithyroid
drug of choice in pregnancy. This approach was supported by
the 2007 Endocrine Society consensus guidelines. It also has
Diagram showing the spectrum of diabetes diagnosis
the advantage of being excreted to a lesser extent than
carbimazole in breast milk.
Management
• lifestyle modification: weight loss, increased exercise, Despite this some endocrinologists use carbimazole and the
change in diet BNF states both drugs may be used in pregnancy.
• at least yearly follow-up with blood tests is recommended Carbimazole has rarely been associated with aplasia cutis of
• NICE recommend metformin for adults at high risk 'whose the neonate
blood glucose measure (fasting plasma glucose or HbA1c)
shows they are still progressing towards type 2 diabetes, PREGNANCY: THYROID PROBLEMS
despite their participation in an intensive lifestyle-change In pregnancy there is an increase in the levels of thyroxine-
programme' binding globulin (TBG). This causes an increase in the levels of
total thyroxine but does not affect the free thyroxine level
Impaired fasting glucose and impaired glucose tolerance
Thyrotoxicosis
There are two main types of IGR:
• impaired fasting glucose (IFG) - due to hepatic insulin Untreated thyrotoxicosis increases the risk of fetal loss,
resistance maternal heart failure and premature labour
• impaired glucose tolerance (IGT) - due to muscle insulin
resistance Graves' disease is the most common cause of thyrotoxicosis in
• patients with IGT are more likely to develop T2DM and pregnancy. It is also recognised that activation of the TSH
cardiovascular disease than patients with IFG receptor by HCG may also occur - often termed transient
gestational hyperthyroidism. HCG levels will fall in second and
Definitions third trimester
• a fasting glucose greater than or equal to 6.1 but less
than 7.0 mmol/l implies impaired fasting glucose (IFG)
Management From a practical point of view patients on insulin who want to
• propylthiouracil has traditionally been the antithyroid apply for a Group 2 (HGV) licence need to complete a VDIAB1I
drug of choice. This approach was supported by the 2007 form.
Endocrine Society consensus guidelines
• maternal free thyroxine levels should be kept in the Other specific points for group 1 drivers:
upper third of the normal reference range to avoid fetal • if on insulin then patient can drive a car as long as they
hypothyroidism have hypoglycaemic awareness, not more than one
• thyrotrophin receptor stimulating antibodies should be episode of hypoglycaemia requiring the assistance of
checked at 30-36 weeks gestation - helps to determine another person within the preceding 12 months and no
risk of neonatal thyroid problems relevant visual impairment. Drivers are normally
• block-and-replace regimes should not be used in contacted by DVLA
pregnancy • if on tablets or exenatide no need to notify DVLA. If
• radioiodine therapy is contraindicated tablets may induce hypoglycaemia (e.g. sulfonylureas)
then there must not have been more than one episode of
Hypothyroidism hypoglycaemia requiring the assistance of another person
within the preceding 12 months
Key points • if diet controlled alone then no requirement to inform
• thyroxine is safe during pregnancy DVLA
• serum thyroid stimulating hormone measured in each
trimester and 6-8 weeks post-partum *to demonstrate adequate control, the Secretary of State's
• some women require an increased dose of thyroxine Honorary Medical Advisory Panel on Diabetes Mellitus has
during pregnancy recommended that applicants will need to have used blood
• breast feeding is safe whilst on thyroxine glucose meters with a memory function to measure and
record blood glucose levels for at least 3 months prior to
Q-4 submitting their application
A middle aged man with type 2 diabetes wants to apply for a
HGV licence. His current treatment is metformin and Q-5
Insulatard (isophane insulin). Which form should you advise A 64-year-old man is reviewed in clinic. He has a history of
him to complete given his insulin therapy? ischaemic heart disease and was diagnosed with type 2
diabetes mellitus around 12 months ago. At this time of
A. D1 diagnosis his HbA1c was 7.6% (60 mmol/mol) and he was
B. SE17 started on metformin which was titrated up to a dose of 1g
C. HGV2-DMI bd. The most recent bloods show a HbA1c of 6.8% (51
D. HGV4 mmol/mol). He has just retired from working in the IT
E. VDIAB1I industry and his body mass index (BMI) today is 28 kg/m².
His other medication is as follows:
ANSWER:
VDIAB1I Atorvastatin 80mg on
Aspirin 75mg od
EXPLANATION: Bisoprolol 2.5 mg od
DVLA: DIABETES MELLITUS Ramipril 5mg od
Until recently people with diabetes who used insulin could not
hold a HGV licence. The DVLA changed the rules in October What is the most appropriate next step?
2011. The following standards need to be met (and also apply
to patients using other hypoglycaemic inducing drugs such as A. Add sitagliptin
sulfonylureas): B. Make no changes to his medication
• there has not been any severe hypoglycaemic event in C. Add glimepiride
the previous 12 months D. Add pioglitazone
• the driver has full hypoglycaemic awareness E. Add exenatide
• the driver must show adequate control of the condition
by regular blood glucose monitoring*, at least twice daily ANSWER:
and at times relevant to driving Make no changes to his medication
• the driver must demonstrate an understanding of the
EXPLANATION:
risks of hypoglycaemia
Since the publication of the 2015 guidelines, NICE
• here are no other debarring complications of diabetes
recommend we only add another drug if the HbA1c has risen
to >= 58 mmol/mol (7.5%) at this stage.
DIABETES MELLITUS: MANAGEMENT OF TYPE 2 • you review a patient 6 months after starting metformin.
NICE updated its guidance on the management of type 2 His HbA1c is 51 mmol/mol (6.8%). You increase his
diabetes mellitus (T2DM) in 2015. Key points are listed below: metformin from 500mg bd to 500mg tds and reinforce
• HbA1c targets have changed. They are now dependent on lifestyle factors
what antidiabetic drugs a patient is receiving and other
factors such as frailty Patient already on treatment
• there is more flexibility in the second stage of treating
patients (i.e. after metformin has been started) - you now Management of T2DM HbA1c target
have a choice of 4 oral antidiabetic agents Already on one drug, but HbA1c has risen to 58 53 mmol/mol (7.0%)
mmol/mol (7.5%)
It's worthwhile thinking of the average patient who is taking
metformin for T2DM, you can titrate up metformin and Drug treatment
encourage lifestyle changes to aim for a HbA1c of 48 The 2015 NICE guidelines introduced some changes into the
mmol/mol (6.5%), but should only add a second drug if the management of type 2 diabetes. There are essentially two
HbA1c rises to 58 mmol/mol (7.5%) pathways, one for patients who can tolerate metformin, and
one for those who can't:
Dietary advice
• encourage high fibre, low glycaemic index sources of
carbohydrates
• include low-fat dairy products and oily fish
• control the intake of foods containing saturated fats and
trans fatty acids
• limited substitution of sucrose-containing foods for other
carbohydrates is allowable, but care should be taken to
avoid excess energy intake
• discourage use of foods marketed specifically at people
with diabetes
• initial target weight loss in an overweight person is 5-10%
HbA1c targets
Practical examples
• you review an established type 2 diabetic on maximum
dose metformin. Her HbA1c is 55 mmol/mol (7.2%). You
do not add another drug as she has not reached the Graphic showing choice of statin.
threshold of 58 mmol/mol (7.5%) *this is a bit confusing because isn't the diagnostic criteria for
• a type 2 diabetic is found to have a HbA1c of 62 T2DM HbA1c 48 mmol/mol (6.5%)? So shouldn't all patients
mmol/mol (7.8%) at annual review. They are currently on be offered metformin at diagnosis? Our interpretation of this
maximum dose metformin. You elect to add a is that some patients upon diagnosis will elect to try lifestyle
sulfonylurea measures, which may reduce their HbA1c below this level. If it
then rises to the diagnostic threshold again metformin should
Cannot tolerate metformin or contraindicated be offered
• if the HbA1c rises to 48 mmol/mol (6.5%)* on lifestyle
interventions, consider one of the following: Q-6
• → sulfonylurea A 67-year-old man presents to the GP with a six month
• → gliptin history of excessive tiredness, feeling thirsty, weight loss and
• → pioglitazone passing a lot of urine. He has a strong family history of Type
• if the HbA1c has risen to 58 mmol/mol (7.5%) then a one 2 diabetes mellitus.
of the following combinations should be used:
• → gliptin + pioglitazone Examination showed blood pressure of 125/72 mmHg and
• → gliptin + sulfonylurea urinalysis showed ++glucose but no ketones. Abdomen was
• → pioglitazone + sulfonylurea soft and tender with no masses. His body mass index is 32
• if despite this the HbA1c rises to, or remains above 58 kg/m². He is retired and does not drive.
mmol/mol (7.5%) then consider insulin therapy
Blood tests arranged showed HbA1c at 58 mmol/L.
Starting insulin
• metformin should be continued. In terms of other drugs Which of the following is an important part of management
NICE advice: 'Review the continued need for other blood of this patient with new onset Type 2 Diabetes Mellitus in
glucose lowering therapies' this age group as per NICE guidelines?
• NICE recommend starting with human NPH insulin
(isophane, intermediate acting) taken at bed-time or A. Referral to secondary care diabetic team for medication
twice daily according to need review
B. CT abdomen
Risk factor modification C. No need to discuss medications but repeat bloods in two
to three months to monitor
Blood pressure D. Start gliclazide and titrate to maximum tolerated dose
E. Issue blood sugar monitoring devices for home checks
• target is < 140/80 mmHg (or < 130/80 mmHg if end-organ
damage is present)
ANSWER:
• ACE inhibitors are first-line
CT abdomen
Antiplatelets
• should not be offered unless a patient has existing
cardiovascular disease
EXPLANATION:
Patients over the age of 60 with new onset diabetes and Conditions where HbA1c may not be used for diagnosis:
weight loss should be referred for an urgent CT abdomen to • haemoglobinopathies
exclude pancreatic cancer • haemolytic anaemia
This patient has a new diagnosis of type 2 diabetes mellitus • untreated iron deficiency anaemia
on the basis of symptoms and blood tests. As he is over 60 • suspected gestational diabetes
and has new onset diagnosis, he should have an urgent CT • children
abdomen performed in order to rule out pancreatic cancer as • HIV
a cause for the new onset diabetes in this age group. There is • chronic kidney disease
no need to refer to secondary care as metformin can be • people taking medication that may cause hyperglycaemia
initiated in primary care. One should consider medications (for example corticosteroids)
given there is enough in the history and bloods to diagnose
type 2 diabetes mellitus - of course, the patient can elect for Impaired fasting glucose and impaired glucose tolerance
lifestyle changes if counselled. Gliclazide is not as suitable as
metformin in this patient, given his body mass index. One A fasting glucose greater than or equal to 6.1 but less than 7.0
would not routinely give type 2 diabetes mellitus patients mmol/l implies impaired fasting glucose (IFG)
blood sugar monitoring devices as Hba1c monitoring is
sufficient - an exception would be if the patient is started on Impaired glucose tolerance (IGT) is defined as fasting plasma
a drug that can cause hypoglycaemia for instance. glucose less than 7.0 mmol/l and OGTT 2-hour value greater
than or equal to 7.8 mmol/l but less than 11.1 mmol/l
Source: NICE CKS
Diabetes UK suggests:
DIABETES MELLITUS (TYPE 2): DIAGNOSIS • 'People with IFG should then be offered an oral glucose
The diagnosis of type 2 diabetes mellitus can be made by tolerance test to rule out a diagnosis of diabetes. A result
either a plasma glucose or a HbA1c sample. Diagnostic criteria below 11.1 mmol/l but above 7.8 mmol/l indicates that
vary according to whether the patient is symptomatic the person doesn't have diabetes but does have IGT.'
(polyuria, polydipsia etc) or not.
Q-7
If the patient is symptomatic: You are reviewing a 57-year-old man who was diagnosed
• fasting glucose greater than or equal to 7.0 mmol/l with type 2 diabetes mellitus around four months ago. At
• random glucose greater than or equal to 11.1 mmol/l (or the time of diagnosis his HbA1c was 54 mmol/mol (7.1%). He
after 75g oral glucose tolerance test) was started on metformin and the dose was titrated up. At
what threshold should you consider adding a second agent?
If the patient is asymptomatic the above criteria apply but
must be demonstrated on two separate occasions. A. 42 mmol/mol (6.0%)
B. 45 mmol/mol (6.3%)
C. 48 mmol/mol (6.5%)
D. 53 mmol/mol (7.0%)
E. 58 mmol/mol (7.5%)
ANSWER:
58 mmol/mol (7.5%)
EXPLANATION:
Diagram showing the spectrum of diabetes diagnosis Please see Q-5 for Diabetes mellitus: management of type 2
In 2011 WHO released supplementary guidance on the use of
Q-8
HbA1c on the diagnosis of diabetes:
You see a 48-year-old man who has recently been diagnosed
• a HbA1c of greater than or equal to 48 mmol/mol (6.5%)
with type 2 diabetes by your GP colleague. The patient has
is diagnostic of diabetes mellitus
been started on gliclazide. He has had one episode of mild
• a HbAlc value of less than 48 mmol/mol (6.5%) does not
hypoglycaemia since starting this medication. The patient
exclude diabetes (i.e. it is not as sensitive as fasting
asks about driving his car. What is the most appropriate
samples for detecting diabetes)
advice?
• in patients without symptoms, the test must be repeated
to confirm the diagnosis A. Must always have a sugary drink in his car
• it should be remembered that misleading HbA1c results B. Must contact DVLA if has two episodes of severe
can be caused by increased red cell turnover (see below) hypoglycaemia in 12 months
C. Must contact the DVLA immediately Features
D. Must take his BM twice daily when driving • features of hypothyroidism
E. Must not drive for four weeks • goitre: firm, non-tender
• anti-thyroid peroxidase and also anti-Tg antibodies
ANSWER:
Must contact DVLA if has two episodes of severe
hypoglycaemia in 12 months
EXPLANATION:
For Group 1 drivers with diabetes taking oral medication
which could cause hypoglycaemia the DVLA do not need to
be notified as long as the patient is under regular review and
has not had more than one episode of hypoglycaemia
requiring the assistance of another person within the past 12
months. Group 2 drivers do need to contact the DVLA and
have stricter requirements. They would be unable to drive if
Venn diagram showing how different causes of thyroid dysfunction may manifest.
they had a single episode of hypoglycaemia require the
Note how many causes of hypothyroidism may have an initial thyrotoxic phase.
assistance of another person within the past 12 months.
Q-10
Both groups must inform the DVLA if they develop and A 43-year-old woman presents for follow-up in clinic. She
impairment to the awareness of hypoglycaemia. was diagnosed with Hashimoto's thyroiditis four months ago
and is currently being treated with levothyroxine 75 mcg od.
Please see Q-4 for DVLA: diabetes mellitus What is the single most important blood test to assess her
response to treatment?
Q-9
A 36-year-old woman presents with feeling tired and cold all A. ESR
the time. On examination a firm, non-tender goitre is noted. B. TSH
Blood tests reveal the following: C. Free T4
D. Total T4
TSH 34.2 mU/l E. Free T3
Free T4 5.4 pmol/l ANSWER:
TSH
What is the most likely diagnosis?
EXPLANATION:
A. Primary atrophic hypothyroidism HYPOTHYROIDISM: MANAGEMENT
B. Pituitary failure Key points
C. De Quervain's thyroiditis • initial starting dose of levothyroxine should be lower in
D. Iodine deficiency elderly patients and those with ischaemic heart disease.
E. Hashimoto's thyroiditis The BNF recommends that for patients with cardiac
disease, severe hypothyroidism or patients over 50 years
Hashimoto's thyroiditis = hypothyroidism + goitre + anti-TPO the initial starting dose should be 25mcg od with dose
The combination of a goitre with hypothyroidism points to a slowly titrated. Other patients should be started on a
diagnosis of Hashimoto's. De Quervain's thyroiditis typically dose of 50-100mcg od
causes a painful goitre. • following a change in thyroxine dose thyroid function
tests should be checked after 8-12 weeks
ANSWER: • the therapeutic goal is 'normalisation' of the thyroid
Hashimoto's thyroiditis stimulating hormone (TSH) level. As the majority of
unaffected people have a TSH value 0.5-2.5 mU/l it is now
EXPLANATION: thought preferable to aim for a TSH in this range
HASHIMOTO'S THYROIDITIS • women with established hypothyroidism who become
Hashimoto's thyroiditis is an autoimmune disorder of the pregnant should have their dose increased 'by at least 25-
thyroid gland. It is typically associated with hypothyroidism 50 micrograms levothyroxine'* due to the increased
although there may be a transient thyrotoxicosis in the acute demands of pregnancy. The TSH should be monitored
phase. It is 10 times more common in women carefully, aiming for a low-normal value
• there is no evidence to support combination therapy with
levothyroxine and liothyronine
79% of Muslim patients with type 2 diabetes mellitus fast
Side-effects of thyroxine therapy Ramadan2.There is an excellent patient information leaflet
• hyperthyroidism: due to over treatment from Diabetes UK and the Muslim Council of Britain which
• reduced bone mineral density explores these options in more detail.
• worsening of angina
• atrial fibrillation If a patient with type 2 diabetes mellitus does decide to fast:
• they should try and and eat a meal containing long-acting
Interactions carbohydrates prior to sunrise (Suhoor)
• iron: absorption of levothyroxine reduced, give at least 2 • patients should be given a blood glucose monitor to allow
hours apart them to check their glucose levels, particularly if they feel
unwell
*source: NICE Clinical Knowledge Summaries • for patients taking metformin the expert consensus is
that the dose should be split one-third before sunrise
Q-11 (Suhoor) and two-thirds after sunset (Iftar)
A 56-year-old Muslim man with a history of type 2 diabetes • expert consensus also recommends switching once-daily
asks for advice. He is due to start fasting for Ramadan soon sulfonylureas to after sunset. For patients taking twice-
and is unsure what he should do with regards to his diabetes daily preparations such as gliclazide it is recommended
medications. He currently takes metformin 500mg tds. What that a larger proportion of the dose is taken after after
is the most appropriate advice? sunset
• no adjustment is needed for patients taking pioglitazone
A. Switch to subcutaneous biphasic insulin for the duration
of Ramadan Q-12 THROUGH 14
B. 500 mg at the predawn meal + 1000 mg at the sunset Theme: Diabetes mellitus: management of type 2
meal
C. No change to the metformin dose A. < 130/80 mmHg
D. 1000 mg at the predawn meal + 500 mg at the sunset B. < 125/75 mmHg
meal C. < 120/70 mmHg
E. Stop metformin for the duration of Ramadan D. 6.5% (48 mmol/mol)
E. 6.0% (42 mmol/mol)
ANSWER: F. 6.2% (44 mmol/mol)
500 mg at the predawn meal + 1000 mg at the sunset meal G. Aspirin
H. Statin
EXPLANATION: I. Ramipril
During Ramadan, one-third of the normal metformin dose J. No additional treatment
should be taken before sunrise and two-thirds should be
taken after sunset For each one of the following select the most appropriate
Please see the Diabetes Care link for more details. answer
ANSWER: ANSWER:
< 130/80 mmHg 6.5% (48 mmol/mol)
Metformin A. 3 monthly
• NICE recommend considering adding metformin if the B. 6 monthly
BMI >= 25 kg/m² C. Once a year
D. Every 2 years
Q-18 E. Only if new problems develop
A 72-year-old man is reviewed in the diabetes clinic. He has a
history of heart failure and type 2 diabetes mellitus. His ANSWER:
current medications include furosemide 40mg od, ramipril 6 monthly
10mg od and bisoprolol 5mg od. Clinical examination is
unremarkable with no evidence of peripheral oedema, a EXPLANATION:
clear chest and blood pressure of 130/76 mmHg. Recent NICE recommend checking the HbA1c every 6 months once
renal and liver function tests are normal. Which one of the treatment is stable in type 2 diabetes mellitus
following medications is contraindicated?
Please see Q-5 for Diabetes mellitus: management of type 2
A. Sitagliptin
B. Pioglitazone Q-20
C. Gliclazide A 23-year-old man is diagnosed as having type 1 diabetes
D. Exenatide mellitus after presenting with diabetic ketoacidosis. His
E. Metformin blood sugars are now stable and he is well. What is the first-
line insulin regime he should be offered?
ANSWER:
Pioglitazone Twice‑daily mixed insulin
Once-daily mixed insulin
EXPLANATION: Basal–bolus insulin regimen with twice-daily insulin detemir
Thiazolidinediones are absolutely contraindicated in heart Basal–bolus insulin regimen with once-daily insulin glargine
failure Rapid‑acting insulin analogue before each meal with no
longer acting insulin
THIAZOLIDINEDIONES
Thiazolidinediones are a class of agents used in the treatment In newly diagnosed adults with type 1 diabetes, the first-line
of type 2 diabetes mellitus. They are agonists to the PPAR- insulin regime should be a basal–bolus using twice‑daily
gamma receptor and reduce peripheral insulin resistance. insulin detemir
Rosiglitazone was withdrawn in 2010 following concerns
about the cardiovascular side-effect profile. ANSWER:
Basal–bolus insulin regimen with twice-daily insulin detemir
The PPAR-gamma receptor is an intracellular nuclear receptor.
It's natural ligands are free fatty acids and it is thought to EXPLANATION:
control adipocyte differentiation and function. Please see Q-17 for Diabetes Mellitus: Management Of Type 1
Q-22
Q-21 A 60-year-old man who is known to have lung cancer comes
A 36-year-old female with a BMI of 34 kg/m^2 presents to for review. For the past three weeks he has lost his appetite,
her GP after managing to lose 3 kg in the past month. She has been feeling sick and generally feels tired. On
asks about the possibility of starting a drug to help her lose examination he appears to be mildly dehydrated. You order
weight. What is the primary mode of action of orilistat? some blood tests:
EXPLANATION: A. Amlodipine
The primary mode of action of orilistat is to inhibit B. Simvastatin
pancreatic lipases, which in turn will decrease the absorption C. Bendroflumethiazide
of lipids from the intestine D. Aspirin
E. Lisinopril
OBESITY: THERAPEUTIC OPTIONS
The management of obesity consists of a step-wise approach: ANSWER:
• conservative: diet, exercise Bendroflumethiazide
• medical
• surgical EXPLANATION:
Thiazides cause hypercalcaemia
Orlistat is a pancreatic lipase inhibitor used in the
management of obesity. Adverse effects include faecal HYPERCALCAEMIA: CAUSES
urgency/incontinence and flatulence. A lower dose version is Two conditions account for 90% of cases of hypercalcaemia:
now available without prescription ('Alli'). NICE have defined 1. Primary hyperparathyroidism: commonest cause in non-
criteria for the use of orlistat. It should only be prescribed as hospitalised patients
part of an overall plan for managing obesity in adults who 2. Malignancy: the commonest cause in hospitalised patients.
have: This may be due to number of processes, including; bone
• BMI of 28 kg/m^2 or more with associated risk factors, or metastases, myeloma, PTHrP from squamous cell lung cancer
• BMI of 30 kg/m^2 or more
• continued weight loss e.g. 5% at 3 months Other causes include
• orlistat is normally used for < 1 year • sarcoidosis*
• vitamin D intoxication
Sibutramine • acromegaly
• withdrawn January 2010 by the European Medicines • thyrotoxicosis
Agency due to an increased risk of cardiovascular events • Milk-alkali syndrome
• centrally acting appetite suppressant (inhibits uptake of • drugs: thiazides, calcium containing antacids
serotonin and noradrenaline at hypothalamic sites that • dehydration
regular food intake) • Addison's disease
• adverse effects include hypertension (monitor blood • Paget's disease of the bone**
pressure and pulse during treatment), constipation,
headache, dry mouth, insomnia and anorexia *other causes of granulomas may lead to hypercalcaemia e.g.
• contraindicated in psychiatric illness, hypertension, IHD, Tuberculosis and histoplasmosis
stroke, arrhythmias
**usually normal in this condition but hypercalcaemia may
Rimonabant, a specific CB1 cannabinoid receptor antagonist, occur with prolonged immobilisation
was withdrawn
in October 2008 after the European Medicines Agency warned
of serious psychiatric problems including suicide
Q-23 incretins to stimulate insulin release. Examples of these
A 59 year old gentleman with capricious type 2 diabetes drugs include sitagliptin, saxagliptin, etc. Rarely DPP-4
mellitus is reviewed in a community diabetic clinic. Despite inhibitors and PPAR- modulators can cause hepatitis,
optimising lifestyle and diet, glycaemic control is still poor on pancreatitis and pancytopaenia. Previous pancreatitis is a
first line oral hypoglycaemic therapy. His diabetic consultant relative contraindication to their use.
decides to start him on dapagliflozin 10mg daily.
Dapagliflozin is a renal SGLT-2 (sodium glucose transporter)
Which of the following is a common side effect of inhibitor which has only recently obtained UK and US
dapagliflozin the patient should be warned about? licensing authority. It causes an increase in glucose excretion
by the kidney to lower serum glucose concentrations. SGLT-2
A. Pancytopaenia is a sodium/glucose co-transporter protein in the nephrons
B. Jaundice proximal tubule which reabsorbs glucose from the renal
C. Increased likelihood of pancreatitis filtrate. These drugs cause heavy urinary glucose loss and
D. Increased likelihood of urinary infections can cause recurrent urinary infections and candidiasis as well
E. Increased risk of ischaemic heart disease as hypoglycaemia, crystalluria and renal failure due to
osmotic diuresis.
ANSWER:
Increased likelihood of urinary infections Please see Q-5 for Diabetes mellitus: management of type 2
A class of oral hypoglycaemic agent known as the For each of the following scenarios please select the most
thiazolidinediones (pioglitazone, etc) also improve glucose likely diagnosis:
processing by increasing upregulation of genes which
augment glucose and fat metabolism. This occurs by Q-24
stimulation of the nuclear signalling protein peroxisome A 64-year-old man recently discharged from hospital
proliferator-activated receptor gamma (PPAR-). following treatment for a pneumonia:
Thiazolidinediones have fallen out of vogue of late due to
concerns about heart failure and ischaemic heart disease. TSH 0.4 mU/l
Features
• hyperthyroidism
• painful goitre
• raised ESR
• globally reduced uptake on iodine-131 scan
Venn diagram showing how different causes of thyroid dysfunction may manifest.
Management Note how many causes of hypothyroidism may have an initial thyrotoxic phase.
• usually self-limiting - most patients do not require
treatment *now referred to as non-thyroidal illness
• thyroid pain may respond to aspirin or other NSAIDs **TSH may be normal in some cases
• in more severe cases steroids are used, particularly if
Q-27
hypothyroidism develops
Which one of the following unwanted effects is most likely
to occur in patients taking gliclazide?
Q-26
A 34-year-old woman presents with palpitations and feeling
A. Peripheral neuropathy
hot all the time. On examination she has a non-tender
B. Cholestasis
goitre. Bloods show the following:
C. Photosensitivity
D. Syndrome of inappropriate ADH secretion
TSH <0.05 mU/l
E. Weight gain
Free T4 22 pmol/l
ANSWER:
ANSWER: Weight gain
Graves’ disease
EXPLANATION:
EXPLANATION: All of the above side-effects may be seen in patients taking
Graves' disease is a much more likely diagnosis than subacute sulfonylureas but weight gain is the most common.
(De Quervain's) thyroiditis which is associated with a tender
goitre and raised ESR. SULFONYLUREAS
EXPLANATION Q-24 THROUGH 26: Sulfonylureas are oral hypoglycaemic drugs used in the
THYROID FUNCTION TESTS management of type 2 diabetes mellitus. They work by
The interpretation of thyroid function tests is usually increasing pancreatic insulin secretion and hence are only
straightforward: effective if functional B-cells are present. On a molecular level
they bind to an ATP-dependent K+(KATP) channel on the cell
Diagnosis TSH Free T4 Notes membrane of pancreatic beta cells.
Thyrotoxicosis (e.g. Graves' Low High In T3 thyrotoxicosis
disease) the free T4 will be
Common adverse effects
normal
• hypoglycaemic episodes (more common with long acting
Primary hypothyroidism High Low
(primary atrophic
preparations such as chlorpropamide)
hypothyroidism) • weight gain
Secondary hypothyroidism Low Low Replacement steroid
therapy is required Rarer adverse effects
prior to thyroxine • syndrome of inappropriate ADH secretion
Sick euthyroid syndrome* Low** Low Common in hospital • bone marrow suppression
inpatients • liver damage (cholestatic)
T3 is particularly low
• peripheral neuropathy
in these patients
Subclinical hypothyroidism High Normal
Sulfonylureas should be avoided in breast feeding and
Poor compliance with thyroxine High Normal
pregnancy
Steroid therapy Low Normal
Q-28
Which one of the following regarding the management of A 33-year-old woman presents with weight loss and
thyroid problems during pregnancy is incorrect? excessive sweating. her partner reports that she is 'on edge'
all the time and during the consultation you notice a fine
A. Maternal free thyroxine levels should be kept in the tremor. Her pulse rate is 96/min. A large, non-tender goitre
upper third of the normal reference range when treating is noted. Examination of her eyes is unremarkable with no
thyrotoxicosis evidence of exophthalmos.
B. Increased levels of thyroxine-binding globulin are seen
in pregnancy Free T4 26 pmol/l
Management Features
• propylthiouracil has traditionally been the antithyroid • typical features of thyrotoxicosis
drug of choice. This approach was supported by the 2007 • specific signs limited to Grave's (see below)
Endocrine Society consensus guidelines
• maternal free thyroxine levels should be kept in the Features seen in Graves' but not in other causes of
upper third of the normal reference range to avoid fetal thyrotoxicosis
hypothyroidism • eye signs (30% of patients): exophthalmos,
• thyrotrophin receptor stimulating antibodies should be ophthalmoplegia
checked at 30-36 weeks gestation - helps to determine • pretibial myxoedema
risk of neonatal thyroid problems • thyroid acropachy
• block-and-replace regimes should not be used in
pregnancy Autoantibodies
• radioiodine therapy is contraindicated • TSH receptor stimulating antibodies (90%)
• anti-thyroid peroxidase antibodies (75%)
Hypothyroidism
Key points Q-30
• thyroxine is safe during pregnancy Which of the following results establishes a diagnosis of
• serum thyroid stimulating hormone measured in each diabetes mellitus?
trimester and 6-8 weeks post-partum
• some women require an increased dose of thyroxine A. Asymptomatic patient with fasting glucose 7.9 mmol/L
during pregnancy on one occasion
• breast feeding is safe whilst on thyroxine B. Symptomatic patient with fasting glucose 6.8 mmol/L on
Q-29 two occasions
C. Glycosuria +++ A 50-year-old man who is known to have obesity and
D. Asymptomatic patient with random glucose 22.0 hypertension comes for review. His current BMI is 38 kg/m^2
mmol/L on one occasion and blood pressure today is 154/92 mmHg despite ramipril
E. Symptomatic patient with random glucose 12.0 mmol/L and bendroflumethiazide. Lifestyle and a trial of orlistat have
on one occasion failed to reduce his weight. Which one of the following is the
most suitable intervention?
ANSWER:
Symptomatic patient with random glucose 12.0 mmol/L on A. Biliopancreatic diversion with duodenal switch
one occasion B. Laparoscopic-adjustable gastric banding
C. Trial of sibutramine
EXPLANATION: D. Referral for counselling to discuss his excessive eating
Diabetes diagnosis: fasting > 7.0, random > 11.1 - if E. Sleeve gastrectomy
asymptomatic need two readings
ANSWER:
Please see Q-6 For Diabetes Mellitus (Type 2): Diagnosis Laparoscopic-adjustable gastric banding
Q-31 EXPLANATION:
A middle-aged man with type 2 diabetes mellitus is Laparoscopic-adjustable gastric banding is the intervention
reviewed. Despite weight loss, metformin and gliclazide his of choice in patients with a BMI < 40 kg/m^2.
HbA1c is 68 mmol/mol (8.4%). The patient agrees to start
insulin therapy. According to NICE guidelines which type of Sibutramine has recently been withdrawn due to concerns
insulin should be tried initially? about a possible increased risk of cardiovascular events.
ANSWER: EXPLANATION:
He may be able to apply for a HGV licence if he meets strict Decisions on primary prevention of cardiovascular disease
criteria relating to hypoglycaemia should be made using standard tools and are not dependant
on whether a diagnosis of metabolic syndrome is made.
EXPLANATION:
Patients on insulin may now hold a HGV licence if they meet METABOLIC SYNDROME
strict DVLA criteria
The April 2009 AKT feedback report made specific mention of Unfortunately there are a number of competing definitions of
fitness to drive rules. the metabolic syndrome around at the present time. It is
Please see Q-4 for DVLA: diabetes mellitus thought that the key pathophysiological factor is insulin
resistance.
Q-34
A 45-year woman who you have treated for obesity comes SIGN recommend using criteria similar to those from the
for review. Despite ongoing lifestyle interventions and trials American Heart Association. The similarity of the International
of orlistat and sibutramine she has failed to lose a significant Diabetes Federation criteria should be noted. For a diagnosis
amount of weight. She is currently taking ramipril for of metabolic syndrome at least 3 of the following should be
hypertension but a recent fasting glucose was normal. For identified:
this patient, what is the cut-off body mass index (BMI) that • elevated waist circumference: men > 102 cm, women >
would trigger a referral for consideration of bariatric 88 cm
surgery? • elevated triglycerides: > 1.7 mmol/L
• reduced HDL: < 1.03 mmol/L in males and < 1.29 mmol/L
A. BMI > 35 kg/m^2 in females
B. BMI > 40 kg/m^2
• raised blood pressure: > 130/85 mmHg, or active
C. BMI > 30 kg/m^2
treatment of hypertension
D. BMI > 38 kg/m^2
• raised fasting plasma glucose > 5.6 mmol/L, or previously
E. BMI > 45 kg/m^2
diagnosed type 2 diabetes
A 67-year-old woman presents with lethargy, depression and
The International Diabetes Federation produced a consensus constipation. A set of screening blood tests reveals the
set of diagnostic criteria in 2005, which are now widely in use. following:
These require the presence of central obesity (defined as
waist circumference > 94cm for Europid men and > 80cm for Calcium 3.05 mmol/l
Europid women, with ethnicity specific values for other Albumin 41 g/l
groups) plus any two of the following four factors:
• raised triglycerides level: > 1.7 mmol/L, or specific
What is the single most useful test for determining the cause
treatment for this lipid abnormality
of her hypercalcaemia?
• reduced HDL cholesterol: < 1.03 mmol/L in males and <
1.29 mmol/L in females, or specific treatment for this lipid
A. ESR
abnormality B. Phosphate
• raised blood pressure: > 130/85 mm Hg, or active C. Vitamin D level
treatment of hypertension D. Parathyroid hormone
• raised fasting plasma glucose > 5.6 mmol/L, or previously E. ACE level
diagnosed type 2 diabetes
ANSWER:
In 1999 the World Health Organization produced diagnostic Parathyroid hormone
criteria which required the presence of diabetes mellitus,
impaired glucose tolerance, impaired fasting glucose or insulin EXPLANATION:
resistance, AND two of the following: Parathyroid hormone levels are useful as malignancy and
• blood pressure: > 140/90 mmHg primary hyperparathyroidism are the two most common
• dyslipidaemia: triglycerides: > 1.695 mmol/L and/or high- causes of hypercalcaemia. A parathyroid hormone that is
density lipoprotein cholesterol (HDL-C) < 0.9 mmol/L normal or raised suggests primary hyperparathyroidism.
(male), < 1.0 mmol/L (female)
• central obesity: waist:hip ratio > 0.90 (male), > 0.85 Please see Q-22 for hypercalcaemia: causes
(female), and/or body mass index > 30 kg/m2
• microalbuminuria: urinary albumin excretion ratio > 20 Q-38
mg/min or albumin:creatinine ratio > 30 mg/g A 56-year-old lady with a BMI of 26 is reviewed in the
diabetic clinic due to poor glycaemic control. She is currently
Other associated features include: being treated with gliclazide 160mg bd. Her latest bloods
• raised uric acid levels show:
• non-alcoholic fatty liver disease
• polycystic ovarian syndrome Na+ 139 mmol/l
K+ 4.1 mmol/l
Q-36
Urea 8.4 mmol/l
A 62-year-old man is reviewed in diabetes clinic. His
glycaemic control is poor despite weight loss, adherence to a Creatinine 180 µmol/l
diabetic diet and his current diabetes medications. He has no ALT 25 iu/l
other past medical history of note. Which one of the
yGT 33 iu/l
following medications would increase insulin sensitivity?
HbA1c 9.4%
A. Repaglinide
B. Tolbutamide Which one of the following medications should be added
C. Pioglitazone next?
D. Acarbose
E. Gliclazide A. Guar gum
B. Pioglitazone
ANSWER: C. Metformin
Pioglitazone D. Acarbose
E. Repaglinide
EXPLANATION:
Please see Q-18 for Thiazolidinediones ANSWER:
Pioglitazone
Q-37
EXPLANATION:
Given that she is overweight metformin would be a natural • If unable to take struggling to eat may need sugary drinks
choice to add. The creatinine however is elevated so to maintain carbohydrate intake
pioglitazone is the next best option. NICE recommend that • It is useful to educate patients so that they have a box of
the metformin dose should be reviewed if the creatinine is > 'sick day supplies' that they can access if they become
130 µmol/l (or eGFR < 45 ml/min) and stopped if the unwell
creatinine is > 150 µmol/l (or eGFR < 30 ml/min) • Access to a mobile phone has been shown to reduce
progression of ketosis to diabetic ketoacidosis
One possible drawback of using pioglitazone is that it may
lead to weight gain, especially given her BMI is already 26. If a patient is taking oral hypoglycaemic medication, they
should be advised to continue taking their medication even if
Please see Q-18 for Thiazolidinediones they are not eating much. Remember that the stress response
to illness increases cortisol levels pushing blood sugars high
Q-39 even without much oral intake. The possible exception is with
A 19-year-old female arrives at your clinic with flu-like metformin, which should be stopped if a patient is becoming
symptoms, she has recently been diagnosed with type 1 dehydrated because of the potential impact upon renal
diabetes and has come for advice regarding her diabetes function.
management whilst she is ill. Which of the following options
is one of the 'sick-day rules' insulin-dependent diabetics If a patient is on insulin, they must not stop it due to the risk
should adhere to during illness? of diabetic ketoacidosis. They should continue their normal
insulin regime but ensure that they are checking their blood
A. Reduce insulin doses sugars frequently. Patients should be able to check their
B. Frequently monitor their blood glucose as the normally ketone levels and if these are raised and blood sugars are also
would raised they may need to give corrective doses of insulin. The
C. Substitute all main meals with sugar-containing foods corrective dose to be given varies by patient, but a rule of
D. Aim to drink at least 3L of fluid thumb would be total daily insulin dose divided by 6
E. Check urinary ketones at the start of illness (maximum 15 units). NHS Scotland have produced a useful
flowsheet for patients to follow:
ANSWER:
Aim to drink at least 3L of fluid Possible indications that a patient might require admission to
hospital would include:
EXPLANATION: • Suspicion of underlying illness requiring hospital
Option 4 is the correct answer as patients should be treatment eg myocardial infarction
encouraged to drink at least 3L of fluid over 24 hours. • Inability to keep fluids down - admit if persisting more
Patients should continue their normal insulin regimen but than a few hours
check their blood glucose more regularly, therefore options 1 • Persistent diarrhoea
and 2 are wrong. Main meals should not be substituted for • Significant ketosis in an insulin dependent diabetic
sugary foods, if a patient is struggling to eat then they may despite additional insulin
take sugary drinks. Ketones should also be measured, but • Blood glucose persistently >20mmol/l despite additional
more frequently than what is being proposed in option 5, for insulin
example, every 3-4 hours or even more frequently depending • Patient unable to manage adjustments to usual diabetes
on the readings. management
• Lack of support at home e.g. a patient who lives alone
NICE Clinical Knowledge Summaries - Diabetes Type 1 and is at risk of becoming unconscious
https://cks.nice.org.uk/diabetes-type-
1#!scenarioclarification:2 Q-40
A diabetic man is diagnosed as having painful diabetic
'Candidates gave generally good responses to questions neuropathy in his feet. He has no other medical history of
concerning management of type 2 diabetes. However, type 1 note. What is the most suitable first-line treatment to relieve
diabetes caused difficulty and candidates are reminded in his pain?
particular to review the management of type 1 diabetes
during intercurrent illness, and sick day rules.' A. Duloxetine
B. Sodium valproate
DIABETES MELLITUS: SICK DAY RULES C. Carbamazepine
The following are key messages that should be given to all D. Referral to pain management clinic
patients with diabetes if they become unwell: E. Tramadol
• Increase frequency of blood glucose monitoring to four
hourly or more frequently
• Encourage fluid intake aiming for at least 3 litres in 24hrs ANSWER:
Duloxetine ANSWER:
Gliptin
EXPLANATION:
DIABETIC NEUROPATHY EXPLANATION:
NICE updated it's guidance on the management of Non-serious gastrointestinal adverse effects of metformin
neuropathic pain in 2013. Diabetic neuropathy is now are common especially when starting the medication. The
managed in the same way as other forms of neuropathic pain: frequency and degree of persistence of adverse effects are
• first-line treatment: amitriptyline, duloxetine, gabapentin dose dependent, but they usually improve with continued
or pregabalin use and are minimised if metformin is started at a low dose,
• if the first-line drug treatment does not work try one of taken with food, and slowly titrated upwards.
the other 3 drugs
• tramadol may be used as 'rescue therapy' for Please see Q-5 for Diabetes mellitus: management of type 2
exacerbations of neuropathic pain
• topical capsaicin may be used for localised neuropathic Q-43
pain (e.g. post-herpetic neuralgia) A 34-year-old woman who takes hydrocortisone and
• pain management clinics may be useful in patients with fludrocortisone replacement therapy for Addison's disease
resistant problems presents for review. She has a three-day history of a
productive cough associated with feeling hot. On
Gastroparesis examination the chest is clear, her pulse is 84 / min and
• symptoms include erratic blood glucose control, bloating temperature is 37.7ºC. You elect to prescribe an antibiotic
and vomiting given her medical history. What is the most appropriate
• management options include metoclopramide, advice with regard to her adrenal replacement therapy?
domperidone or erythromycin (prokinetic agents)
A. Keep the same hydrocortisone and fludrocortisone dose
Q-41 B. Double both the hydrocortisone and fludrocortisone
A diabetic patient who uses insulin presents to the surgery. dose
Which one of the following statements regarding the C. Double the hydrocortisone dose, keep the same
application for a group 2 (HGV) driving licence is correct? fludrocortisone dose
D. Convert her to prednisolone for the duration of the
A. Applicants must not be coprescribed metformin illness
B. The GP needs to complete a D1 form E. Stop the hydrocortisone and fludrocortisone until the
C. A glucose tolerance test should be performed prior to patient recovers
the application
D. Applicants must have had no more than one severe ANSWER:
hypoglycaemic episodes in the past 12 months Double the hydrocortisone dose, keep the same
E. Applicants must provide 3 months of blood glucose fludrocortisone dose
readings using an electronic monitor
EXPLANATION:
ANSWER: Addison's patient unwell? Double the glucocorticoids
Applicants must provide 3 months of blood glucose readings
using an electronic monitor ADDISON'S DISEASE: MANAGEMENT
EXPLANATION:
Please see Q-4 for DVLA: diabetes mellitus Patients who have Addison's disease are usually given both
glucocorticoid and mineralocorticoid replacement therapy.
Q-42
A 56 year old man with type 2 diabetes has had a recent This usually means that patients take a combination of:
HbA1c of 60 mmol/mol. He is currently on the maximum • hydrocortisone: usually given in 2 or 3 divided doses.
dose of gliclazide as he cannot tolerate metformin of any Patients typically require 20-30 mg per day, with the
formulation due to side effects of frequent loose stools. majority given in the morning dose
What further management should you recommend? • fludrocortisone
If a ACTH stimulation test is not readily available (e.g. in EXPLANATION Q-48 THROUGH 50:
primary care) then sending a 9 am serum cortisol can be Please see Q-5 for Diabetes mellitus: management of type 2
useful:
• > 500 nmol/l makes Addison's very unlikely Q-51
• < 100 nmol/l is definitely abnormal A 24-year-old man is presecribed an extended course of oral
• 100-500 nmol/l should prompt a ACTH stimulation test to prednisolone following a flare of ulcerative colitis. Which
be performed one of the following side-effects is most associated with
prolonged corticosteroid use?
Associated electrolyte abnormalities are seen in around one-
third of undiagnosed patients: A. Insomnia
• hyperkalaemia B. Thrombocytopaenia
• hyponatraemia C. Hypotension
• hypoglycaemia D. Bronchospasm
• metabolic acidosis E. Hyperkalaemia
A. Normal EXPLANATION:
B. Diabetes mellitus Psychiatric problems are common with longer term steroid
C. Impaired fasting glucose use
D. Samples mixed up
E. Impaired glucose tolerance CORTICOSTEROIDS
F. Suggests diabetes mellitus but further testing needed Corticosteroids are amongst the most commonly prescribed
G. Impaired fasting glucose and impaired glucose tolerance therapies in clinical practice. They are used both systemically
(oral or intravenous) or locally (skin creams, inhalers, eye
Please select the diagnosis for each of the following drops, intra-articular). They augment and in some cases
scenarios: replace the natural glucocorticoid and mineralocorticoid
activity of endogenous steroids.
Q-48
After fasting overnight a patients urine sample shows The relative glucocorticoid and mineralocorticoid activity of
glucose ++, no ketones commonly used steroids is shown below:
EXPLANATION:
Selected points on the use of corticosteroids: MODY is inherited in an autosomal dominant fashion so a
family history is often present
• patients on long-term steroids should have their doses
doubled during intercurrent illness
MODY
• the BNF suggests gradual withdrawal of systemic
Maturity-onset diabetes of the young (MODY) is characterised
corticosteroids if patients have: received more than 40mg
by the development of type 2 diabetes mellitus in patients <
prednisolone daily for more than one week, received
25 years old. It is typically inherited as an autosomal dominant
more than 3 weeks treatment or recently received
condition. Over six different genetic mutations have so far
repeated courses
been identified as leading to MODY.
Q-52
It is thought that around 1-2% of patients with diabetes
A 62-year-old gentleman with a background of myocardial
mellitus have MODY, and around 90% are misclassified as
infarction, congestive heart failure and chronic obstructive
having either type 1 or type 2 diabetes mellitus.
pulmonary disease attends for a diabetes review at his GP.
He has recently been diagnosed with type 2 diabetes
MODY 3
mellitus and despite a trial of lifestyle modifications his
• 60% of cases
HbA1c is 56 mmol/mol. His GP decides to commence drug
treatment. • due to a defect in the HNF-1 alpha gene
Please see Q-5 for Diabetes mellitus: management of type 2 A. Liver function tests
B. Full blood count
Q-53 C. Prothrombin time
A 36-year-old woman is reviewed 3 months post-partum. D. Urea and electrolytes
During the pregnancy she was diagnosed with gestational E. Cortisol
diabetes. Following the delivery her glycaemic control has
failed to improve and she has been diagnosed as having type ANSWER:
2 diabetes mellitus. She is only slightly overweight (body Full blood count
146/78 mmHg and 148/76 mmHg. His current medications
EXPLANATION: include metformin and allopurinol. What is the most suitable
Agranulocytosis is associated with carbimazole use and management of his blood pressure readings?
needs to be excluded
A. Stop the allopurinol
CARBIMAZOLE B. Start therapy if 10-year cardiovascular risk is greater
Carbimazole is used in the management of thyrotoxicosis. It is than 20%
typically given in high doses for 6 weeks until the patient C. Continue lifestyle measures and review in 6 months
becomes euthyroid before being reduced. D. Start amlodipine
E. Start an ACE inhibitor
Mechanism of action
• blocks thyroid peroxidase from coupling and iodinating ANSWER:
the tyrosine residues on thyroglobulin → reducing thyroid Start an ACE inhibitor
hormone production
• in contrast propylthiouracil as well as this central EXPLANATION:
mechanism of action also has a peripheral action by Whilst his diastolic is below the target (140 / 80 mmHg) his
inhibiting 5'-deiodinase which reduces peripheral systolic is consistently above. Clinical Knowledge Summaries
conversion of T4 to T3 advise that primary care decision making in Type 2 diabetes
should be based on the systolic value. As this man has type 2
Adverse effects diabetes mellitus the target blood pressure should be <
• agranulocytosis 140/80 mmHg. Lifestyle measures have failed to bring his
• crosses the placenta, but may be used in low doses during blood pressure down so he should be offered an ACE
pregnancy inhibitor.
Q-55 The April 2010 AKT feedback report stated: 'Items concerning
You review a 49-year-old woman who has recently been routine management of type 2 diabetes caused some
diagnosed with rheumatoid arthritis. Around three months problem, especially with regard to the wider management of
ago she was started on methotrexate with the addition of cardiovascular risks.'
prednisolone to gain rapid control of her symptoms. She is
now taking methotrexate 15 mg once weekly and is still Please see Q-5 for Diabetes mellitus: management of type 2
taking prednisolone 10 mg od. Unfortunately she is
experiencing a number of side-effects. Which one of the Q-57
following is most likely to be secondary to the prednisolone? A 65-year-old man who is known to have type 2 diabetes
mellitus presents for advice. He is a Muslim and is
A. 'Loss of appetite' considering fasting for Ramadan. His diabetes is currently
B. 'Tired all the time' controlled with a combination of diet and metformin 500mg
C. 'My shoulder and leg muscles feel weak' tds. Looking at his records the last HbA1c was 6.4% (46
D. 'Diarrhoea' mmol/mol). If he decides to fast during Ramadan, what is
E. 'Blue tinge to my vision' the most appropriate advice to give regarding his
metformin?
ANSWER:
'My shoulder and leg muscles feel weak' A. Metformin should be stopped
B. Metformin 1.5 g after sunset
EXPLANATION: C. Metformin 500 mg before sunrise, 1 g after sunset
Proximal myopathy is common with longer term steroid use. D. Metformin 500 mg after sunset
Some of the other side-effects may of course be secondary to E. Metformin 1g before sunrise, 500 mg after sunset
either the methotrexate or ongoing rheumatoid disease.
ANSWER:
Please see Q-51 for Corticosteroids Metformin 500 mg before sunrise, 1g after sunset
Q-56 EXPLANATION:
A 72-year-old man presents for review. His medical history During Ramadan, one-third of the normal metformin dose
includes type 2 diabetes mellitus and gout. Six months ago should be taken before sunrise and two-thirds should be
his blood pressure was 144/84 mmHg. You gave him basic taken after sunset
lifestyle advice and advised him to come back six months
later for a repeat blood pressure check. Three further blood Please see Q-11 for Diabetes Mellitus: Ramadan
pressure readings have been as follows: 144/72 mmHg,
Q-58
A 28 year old Welsh woman attends her GP with a month Secondary causes
history of fatigue and depression. Her blood pressure is • pituitary disorders (e.g. tumours, irradiation, infiltration)
124/80 mmHg lying and 92/58 mmHg standing. Blood tests
reveal a mild hyponatraemia and mild hyperkalaemia. The Exogenous glucocorticoid therapy
GP arranges a short synacthen test and the results are as
follows: (expected 30 minute level >580 nmol/l) Q-59
A 20-year-old woman who has type 1 diabetes mellitus is
Baseline cortisol 300 nmol/l found collapsed in the corridor. A nurse is already with her
30 minute cortisol 350 nmol/L and has done a finger-prick glucose which is 1.8 mmol/l. On
assessment you find that she is not responsive to voice,
What is the most likely underlying cause? pulse 84/min. The nurse has already placed the patient in
the recovery position. What is the most appropriate next
A. Waterhouse Friderichsen syndrome step in management?
B. Tuberculosis
C. Metastatic disease A. Smear quick-acting carbohydrate gel on the gums
D. Autoimmune adrenalitis B. Give rectal dextrose
E. Amyloidosis C. Give intramuscular protamine sulphate
D. Give intramuscular glucagon
ANSWER: E. Give intramuscular dextrose
Autoimmune adrenalitis
ANSWER:
EXPLANATION: Give intramuscular glucagon
This woman has Addison's disease as she has failed a short
synacthen test. Synacthen is a synthetic adrenocorticotrophic EXPLANATION:
hormone (ACTH) analogue which should stimulate cortisol It is potentially dangerous to place anything inside the
secretion from the adrenal glands. mouth of an unconscious patient as they may not be
protecting their airway properly.
The most common cause of Addison's disease in
industrialised nations is autoimmune disease. Tuberculosis Protamine sulphate is used in heparin overdose.
(TB) is the most common cause worldwide but given the
patient's Welsh ethnicity and the absence of any TB risk INSULIN THERAPY: SIDE-EFFECTS
factors it is less likely in this case. Metastatic disease,
amyloidis and Waterhouse Friderichsen syndrome are all less Hypoglycaemia
common causes of Addison's disease. • patients should be taught the signs of hypoglycaemia:
sweating, anxiety, blurred vision, confusion, aggression
ADDISON'S DISEASE • conscious patients should take 10-20g of a short-acting
Autoimmune destruction of the adrenal glands is the carbohydrate (e.g. a glass of Lucozade or non-diet drink,
commonest cause of primary hypoadrenalism in the UK, three or more glucose tablets, glucose gel)
accounting for 80% of cases • every person treated with insulin should have a glucagon
kit for emergencies where the patient is not able to orally
Features ingest a short-acting carbohydrate
• lethargy, weakness, anorexia, nausea & vomiting, weight • patients who have frequent hypoglycaemic episodes may
loss, 'salt-craving' develop reduced awareness. If this develops then
• hyperpigmentation (especially palmar creases), vitiligo, allowing glycaemic control to slip for a period of time may
loss of pubic hair in women, hypotension restore their awareness
• crisis: collapse, shock, pyrexia • beta-blockers reduce hypoglycaemic awareness
EXPLANATION: ANSWER:
Causes of raised prolactin - the p's Test sensation using a 10 g monofilament
A. Start levothyroxine
EXPLANATION: B. Start carbimazole
A 10 g monofilament should be used to assess for diabetic C. Order a thyroid ultrasound scan
neuropathy in the feet D. Start levothyroxine + carbimazole ('block and replace')
E. Repeat TFTs in a few months time
DIABETIC FOOT DISEASE
Diabetic foot disease is an important complication of diabetes ANSWER:
mellitus which should be screen for on a regular basis. NICE Repeat TFTs in a few months time
produced guidelines relating to diabetic foot disease in 2015.
EXPLANATION:
It occurs secondary to two main factors: This patient has subclinical hypothyroidism. By both the TSH
neuropathy: resulting in loss of protective sensation (e.g. not and age criteria advocated by NICE Clinical Knowledge
noticing a stone in the shoe), Charcot's arthropathy, dry skin Summaries she should be monitored for now.
peripheral arterial disease: diabetes is a risk factor for both
macro and microvascular ischaemia SUBCLINICAL HYPOTHYROIDISM
Presentations Basics
neuropathy: loss of sensation • TSH raised but T3, T4 normal
ischaemia: absent foot pulses, reduced ankle-brachial • no obvious symptoms
pressure index (ABPI), intermittent claudication
complications: calluses, ulceration, Charcot's arthropathy, Significance
cellulitis, osteomyelitis, gangrene • risk of progressing to overt hypothyroidism is 2-5% per
year (higher in men)
All patients with diabetes should be screened for diabetic foot • risk increased by the presence of thyroid autoantibodies
disease on at least an annual basis
screening for ischaemia: done by palpating for both the Management
dorsalis pedis pulse and posterial tibial artery pulse
screening for neuropathy: a 10 g monofilament is used on Not all patients require treatment. NICE Clinical Knowledge
various parts of the sole of the foot Summaries (CKS) have produced guidelines. Note that not all
patients will fall within the age boundaries given and hence
NICE recommend that we risk stratify patients: these are guidelines in the broader sense.
Low risk Moderate risk High risk TSH is between 4 - 10mU/L and the free thyroxine level is
• no risk factors • deformity or • previous ulceration or within the normal range
except callus alone • neuropathy or • previous amputation or • if < 65 years with symptoms suggestive of
• non-critical limb • on renal replacement therapy or
hypothyroidism, give a trial of levothyroxine. If there is no
ischaemia. • neuropathy and non-critical
limb ischaemia together or improvement in symptoms, stop levothyroxine
• neuropathy in combination with • 'in older people (especially those aged over 80 years)
callus and/or deformity or follow a 'watch and wait' strategy, generally avoiding
• non-critical limb ischaemia in hormonal treatment'
combination with callus and/or • if asymptomatic people, observe and repeat thyroid
deformity.
function in 6 months
All patients who are moderate or high risk (I.e. any problems TSH is > 10mU/L and the free thyroxine level is within the
other than simple calluses) should be followed up regularly by normal range
the local diabetic foot centre. • start treatment (even if asymptomatic) with
Q-68 levothyroxine if <= 70 years
An 85-year-old comes for review. She has recently had • 'in older people (especially those aged over 80 years)
private health screening and has been advised to see a follow a 'watch and wait' strategy, generally avoiding
doctor regarding her thyroid function tests (TFTs). hormonal treatment'
EXPLANATION: Q-71
Amitriptyline would normally be first choice but given his A 60-year old woman presents to your surgery worried that
history of benign prostatic hyperplasia it is better to avoid she may be diabetic. She is obese and has a strong family
amitriptyline due to the risk of urinary retention. history of type 2 diabetes. Testing the HbA1c is
inappropriate because she has chronic kidney disease so you
Please see Q-40 for Diabetic Neuropathy decide to perform an oral glucose tolerance test. The result
shows that she has impaired glucose tolerance. Which of the
Q-70 following results would demonstrate this?
A 57-year-old man with a history of type 2 diabetes mellitus
and chronic heart failure is reviewed in the diabetes clinic. A. Fasting plasma glucose = 5.2mmol/L, two hour oral
His current medication list is as follows: glucose tolerance test = 7.4mmol/L
B. Fasting plasma glucose = 5.0mmol/L, two hour oral
metformin 1g bd glucose tolerance test = 7.2mmol/L
gliclazide 160mg bd C. Fasting plasma glucose = 7.5mmol/L, two hour oral
ramipril 10mg od glucose tolerance test = 11.5mmol/L
bisoprolol 5mg od D. Fasting plasma glucose = 5.5mmol/L, two hour oral
furosemide 40mg od glucose tolerance test = 9.8mmol/L,
simvastatin 20mg on E. Fasting plasma glucose = 7.6mmol/L, two hour oral
glucose tolerance test = 11.2mmol/L
His annual bloods show the following:
ANSWER:
Na +
140 mmol/l
Fasting plasma glucose = 5.5mmol/L, two hour oral glucose
tolerance test = 9.8mmol/L,
K +
3.9 mmol/l
ANSWER:
Q-72 Q-74
A 67-year-old man with a history of ischaemic heart disease A 19-year-old male has optimised his behavioural changes
and type 2 diabetes mellitus is noted to have non-visible for weight loss, including diet and exercise. He has lost 12kg
haematuria during an annual review. He is currently feeling over the last 6 months but his BMI (body mass index) today
well and is asymptomatic. The urine dipstick showed blood still measures 32 kg/m^2. He has no other medical problems
++, with no protein and no leucocytes. This result is repeated and is not taking any regular medication. He is worried that
one week later. his extra weight poses a risk to his health. He asks if you
could give him medication to help his weight loss.
His current medications include aspirin, bisoprolol,
atorvastatin, ramipril, metformin and pioglitazone. A. An appetite suppressant such as sibutramine can be
trialled for 3 months and continued if effective
Which one of the following drugs should be stopped whilst B. You could consider orlistat
awaiting further investigations? C. Pharmacological treatment is not advised below a BMI
(body mass index) of 35 kg/m^2
A. Aspirin D. He is a strong candidate for bariatric surgery and you
B. Ramipril should offer him a referral for this if he wishes
C. Atorvastatin E. Start oral thyroxine (T4) to increase his basal metabolic
D. Metformin rate
E. Pioglitazone
ANSWER:
ANSWER: You could consider orlistat
Pioglitazone
EXPLANATION:
EXPLANATION: See explanation below defining the NICE criteria for the use
Pioglitazone has been linked to the development of bladder of orlistat
cancer.
Pharmacological appetite suppressants and oral thyroxine
Please see Q-18 for Thiazolidinediones have not been shown to be safe or effective in clinical trials.
EXPLANATION:
Do a fasting glucose sample when a HbA1c is not Prior to her recent admission the TSH has been within range
appropriate for diagnosing T2DM for the past two years. Which one of the following new drugs
most likely explains the raised TSH?
Please see Q-6 for Diabetes Mellitus (Type 2): Diagnosis
A. Simvastatin
Q-76 B. Clopidogrel
You are reviewing the blood results of a 45-year-old obese C. Ferrous sulphate
man who had been complaining of tiredness. His full blood D. Ramipril
count, urea and electrolytes and thyroid function tests were E. Lansoprazole
within normal limits. The fasting plasma glucose result is
shown below: ANSWER:
Ferrous sulphate
Fasting plasma glucose 6.2 mmol/l
EXPLANATION:
What is the most appropriate interpretation of this result? Iron reduces the absorption of thyroxine
A. Consistent with diabetes mellitus - need to confirm with Please see Q-10 for Hypothyroidism: Management
a repeat sample
B. Normal - no further action needed Q-78
C. Borderline - repeat sample in 12 months A 60-year-old woman who is known to have metastatic
D. Impaired glucose tolerance - moderate risk of breast cancer presents following a grand mal seizure at
developing type 2 diabetes mellitus home. For the past few weeks she has been having
E. Prediabetes - high risk of developing type 2 diabetes progressively worsening headaches. Given the likely
mellitus diagnosis, what is the most appropriate first-line
management whilst she is awaiting brain imaging?
ANSWER:
Prediabetes - high risk of developing type 2 diabetes mellitus A. Prednisolone
B. Paracetamol
EXPLANATION: C. Diazepam
This patient has impaired fasting glycaemia is diagnosed D. Fluid restriction
with a fasting plasma glucose (FPG) between 6.1-6.9 mmol/l E. Dexamethasone
and he should therefore be managed as having prediabetes.
ANSWER:
Please see Q-6 for Diabetes Mellitus (Type 2): Diagnosis Dexamethasone
Q-77 EXPLANATION:
A 75-year-old woman with a history of hypothyoidism is This woman is likely to have cerebral metatases. The first-
admitted to the Emergency Department following an line treatment is high-dose dexamethasone which may
episode of chest pain. She is diagnosed as having an acute reduce cerebral oedema. There may also be a role for anti-
coronary syndrome and iron-deficiency anaemia. A epileptics such as phenytoin to reduce the frequency of
percutaneous coronary intervention is performed and a seizures.
coronary artery stent is inserted. Endoscopies of the upper
and lower gastrointestinal tract are performed and reported Please see Q-51 for Corticosteroids
as normal. She is discharged on the following drugs in
addition to her regular levothyroxine: aspirin, clopidogrel, Q-79
ramipril, lansoprazole, simvastatin and ferrous sulphate. Six A 32-year-old patient is reviewed three months after
weeks later she complains of feeling tired all the time. Her commencing orlistat for management of obesity. Her weight
GP arranges some routine blood tests: prior to starting the medication was 87kg; it is now 80kg,
representing a percentage weight loss of 8%. According to
Hb 11.9 g/dl recent NICE Guidelines published in 2014, what is the usual
minimum percentage weight loss that patients should
Platelets 155 * 109/l
achieve over the first three months to justify continuing
WBC 5.2 * 109/l orlistat therapy?
A. No set criteria holistic clinical decision You see a 42 year-old gentleman who presents feeling tired
B. 2% all the time. You ask if he has been on holiday because he
C. 5% appears tanned, but he says he has not been in the sun. On
D. 10% examination the palmar creases and buccal mucosa show
E. 15% pigmentation. What underlying condition might cause this
presentation?
ANSWER:
5% A. Hyperparathyroidism
B. Renal failure
EXPLANATION: C. Addison's disease
The AKT summary report for the October 2014 exam sitting D. Type 2 diabetes
noted that there seemed to be lack of knowledge around E. Wilson's disease
current guidance on the management of obesity. Please see
the link below for the full NICE guidance on obesity, updated ANSWER:
in 2014. Addison's disease
0 5.9
Q-82
A 54-year-old man with type 2 diabetes mellitus is reviewed
2 8.4
in clinic. He is currently taking pioglitazone, metformin,
aspirin and simvastatin. Which one of the following
problems is most likely to be caused by pioglitazone?
Time (hours) Blood glucose (mmol/l)
0 5.9 A. Photosensitivity
2 8.4 B. Thrombocytopaenia
C. Myalgia
How should these results be interpreted? D. Peripheral oedema
E. Hyponatraemia
A. Impaired fasting glucose and impaired glucose tolerance
B. Normal ANSWER:
C. Diabetes mellitus Peripheral oedema
D. Impaired glucose tolerance
E. Impaired fasting glucose EXPLANATION:
Please see Q-18 for Thiazolidinediones
ANSWER:
Impaired glucose tolerance Q-83
A 40-year-old woman is diagnosed as having Addison's
EXPLANATION: disease. What combination of medications is she most likely
Please see Q-5 for Diabetes mellitus: management of type 2 to be prescribed?
A. Prednisolone + fludrocortisone
Q-81
B. Hydrocortisone + fludrocortisone ANSWER:
C. Hydrocortisone + dehydroepiandrosterone (DHEA) Graves’ disease
D. Prednisolone + spironolactone
E. Hydrocortisone + oestrogen/progesterone EXPLANATION:
The thyrotoxic symptoms and blood tests, goitre and anti-
ANSWER: thyroid peroxidase antibodies suggest a diagnosis of Graves'
Hydrocortisone + fludrocortisone disease.
Q-85 Which of the following is the most likely cause for her
A 44-year-old woman presents to her GP as she is feeling symptoms?
'hot all the time' and is consequently worried she is going
through an early menopause. Her husband has also noticed a A. Primary hyperparathyroidism
'fullness' of her neck which has become apparent over the B. Secondary hyperparathyroidism
past few weeks. On examination her pulse is 90/minute and C. Sarcoidosis
she has a small, non-tender goitre. Blood tests are arranged: D. Tertiary hyperparathyroidism
E. Type 1 renal tubular acidosis
TSH < 0.05 mu/l
ANSWER:
Free T4 24 pmol/l
Primary hyperparathyroidism
Anti-thyroid peroxidase antibodies 102 IU/mL (< 35 IU/mL)
ESR 23 mm/hr
EXPLANATION:
The most likely diagnosis here is primary
hyperparathyroidism caused by parathyroid adenoma or
What is the most likely diagnosis?
hyperplasia. The classical biochemical findings are a high
serum calcium and low phosphate. The parathyroid hormone
A. Hashimoto's thyroiditis
level is either high or inappropriately normal.
B. Toxic multinodular goitre
C. Thyroid cancer
Secondary hyperparathyroidism is caused by chronic
D. De Quervain's thyroiditis
hypocalcaemia (e.g. chronic kidney disease). Serum calcium
E. Graves' disease
is low or normal which parathyroid normal levels are high.
Tertiary hyperparathyroidism develops from secondary
hyperparathyroidism and results in autonomous parathyroid
production. It is usually seen patients with end-stage renal • tricyclics
disease. • isoniazid
• calcium channel blockers
Sarcoidosis and type 1 renal tubular acidosis are rare causes • heroin
of hypercalcaemia. • busulfan
• methyldopa
Please see Q-64 for Primary Hyperparathyroidism
Q-88
Q-87 A 52-year-old man presents to his GP as he is concerned
You review a 70-year-old who has a long past medical history about a discharge from his nipples. Which one of the
and is on multiple drugs. He has developed excessive following drugs is most likely to be responsible?
amounts of breast tissue bilaterally. Which one of the
following drugs is most likely to be responsible? A. Ranitidine
B. Isoniazid
A. Tamoxifen C. Digoxin
B. Terbinafine D. Spironolactone
C. Amiodarone E. Chlorpromazine
D. Goserelin (Zoladex)
E. Lymecycline ANSWER:
Chlorpromazine
ANSWER:
Goserelin (Zoladex) EXPLANATION:
Each of the other four drugs may be associated with
EXPLANATION: gynaecomastia rather than galactorrhoea.
Goserelin is a gonadorelin analogue used in the treatment of
advanced prostate cancer. Tamoxifen may be used to treat Please see Q-66 for Prolactin and Galactorrhoea
gynaecomastia.
Q-89
GYNAECOMASTIA A 53-year-old female with a history of primary atrophic
Gynaecomastia describes an abnormal amount of breast hypothyroidism is assessed two months following a change
tissue in males and is usually caused by an increased in her dose of levothyroxine. Which one of the following best
oestrogen:androgen ratio. It is important to differentiate the describes what the TSH should ideally be?
causes of galactorrhoea (due to the actions of prolactin on
breast tissue) from those of gynaecomastia A. Between 0.5 to 1.0 mU/l
B. Between 0.5 to 2.5 mU/l
Causes of gynaecomastia C. Between 2.5 to 4.5 mU/l
• physiological: normal in puberty D. Between 1.5 to 3.5 mU/l
• syndromes with androgen deficiency: Kallman's, E. Between 3.5 to 5.5 mU/l
Klinefelter's
• testicular failure: e.g. mumps ANSWER:
• liver disease Between 0.5 to 2.5 mU/l
• testicular cancer e.g. seminoma secreting hCG
• ectopic tumour secretion EXPLANATION:
• hyperthyroidism A TSH value between 0.5 to 2.5 mU/l is now considered
• haemodialysis preferable. Dosage changes should of course also take
• drugs: see below account of symptoms
ANSWER:
Stop the prednisolone with no further doses
EXPLANATION:
The BNF suggests gradual withdrawal of systemic
corticosteroids if patients have:
• received more than 40mg prednisolone daily for more
than one week
Which drug is most likely to be responsible?
• received more than 3 weeks treatment
A. Doxycycline • recently received repeated courses
B. Amiodarone
C. Prednisolone Please see Q-51 for Corticosteroids
D. Testosterone replacement therapy
E. Phenytoin Q-93
A 33-year-old woman who is known to have type 1 diabetes
ANSWER: mellitus is reviewed. Her HbA1c control is currently within
Prednisolone target although you note that she regularly misses her
hospital diabetes appointments. How often should she have
EXPLANATION: HbA1c monitoring?
A. Every 6 - 8 weeks A 43-year-old man presents to surgery with lethargy.
B. Every 3 - 6 months Examination is unremarkable apart from a blood pressure of
C. Every 7 - 9 months 192/112 mmHg. Routine blood tests reveal:
D. Every 12 months
E. Every 18 months Na+ 146 mmol/l
K+ 2.4 mmol/l
In type 1 diabetes, the HbA1c should be measured every 3-6
Bicarbonate 34 mmol/l
months
Urea 5.3 mmol/l
Q-95
EXPLANATION:
It is well documented that radioiodine therapy can
precipitate thyroid eye disease but a majority of patients
eventually require thyroxine replacement
ATD titration
• carbimazole is started at 40mg and reduced gradually to
CT abdomen showing a right-sided adrenal adenoma in a patient who presented
maintain euthyroidism
with hypertension and hypokalaemia. The adenoma can be seen 'next to' or
'below' the liver. • typically continued for 12-18 months
• patients following an ATD titration regime have been
Q-96 shown to suffer fewer side-effects than those on a block-
For patients who fast during Ramadan, which one of the and-replace regime
following statements regarding the management of type 2
diabetes mellitus is true? Block-and-replace
• carbimazole is started at 40mg
A. Patients taking metformin should have their doses • thyroxine is added when the patient is euthyroid
halved during Ramadan • treatment typically lasts for 6-9 months
B. Sulfonylureas should be stopped during Ramadan
C. Around 4 out of 5 patients Muslim patients with type 2 The major complication of carbimazole therapy is
diabetes mellitus fast during Ramadan agranulocytosis
D. Short-acting carbohydrates should be taken after
breaking the fast in the evening Radioiodine treatment
E. Pioglitazone should be stopped during Ramadan • contraindications include pregnancy (should be avoided
for 4-6 months following treatment) and age < 16 years.
ANSWER: Thyroid eye disease is a relative contraindication, as it
Around 4 out of 5 patients Muslim patients with type 2 may worsen the condition
diabetes mellitus fast during Ramadan • the proportion of patients who become hypothyroid
depends on the dose given, but as a rule the majority of
EXPLANATION: patient will require thyroxine supplementation after 5
Research suggests that the vast majority (around 80%) of years
Muslims with diabetes will fast during Ramadan
Q-98
Please see Q-11 for Diabetes Mellitus: Ramadan A 40-year-old woman complains of feeling tired all the time
and putting on weight. On examination a diffuse, non-tender
Q-97 goitre is noted. Blood tests are ordered:
A 33-year-old female is referred by her GP with
thyrotoxicosis. Following a discussion of management TSH 15.1 mU/l
options she elects to have radioiodine therapy. Which one of Free T4 7.1 pmol/l
the following is the most likely adverse effect? ESR 14 mm/hr
C. Agranulocytosis
D. Oesophagitis What is the most likely diagnosis?
E. Precipitation of thyroid eye disease
A. Pituitary failure
B. Primary atrophic hypothyroidism
ANSWER:
C. De Quervain's thyroiditis
Hypothyroidism
D. Hashimoto's thyroiditis
E. Grave's disease
It can sometimes seem daunting to understand the various
types of insulin but it is important you have a basic grasp to
ANSWER: avoid potential harm to patients.
Hashimoto's thyroiditis
Classification of insulin
EXPLANATION: By manufacturing process
Hashimoto's thyroiditis = hypothyroidism + goitre + anti-TPO • porcine: extracted and purified from pig pancreas
This patient has Hashimoto's thyroiditis, as evidenced by the • human sequence insulin: either produced by enzyme
hypothyroidism, goitre and anti-thyroid peroxidase modification of porcine insulin (emp) or biosynthetically
antibodies. De Quervain's thyroiditis typically causes a by recombinant DNA using bacteria (crb, prb) or yeast
painful goitre and a raised ESR. Around 90% of patients with (pyr)
Grave's disease have anti-TSH receptor stimulating • analogues
antibodies.
By duration of action
Please see Q-9 for Hashimoto’s thyroiditis Onset Peak Duration
Rapid-acting insulin analogues 5 mins 1 hour 3-5 hours
Q-99 Short-acting insulin 30 mins 3 hours 6-8 hours
You are a GP who is visiting a nursing home. A new patient is Intermediate-acting insulin 2 hours 5-8 hours 12-18 hours
admitted and the nurse is looking through his medications Long-acting insulin analogues 1-2 hours Flat profile Up to 24 hours
which he has brought with him from home. The patient is Premixed preparations - - -
diabetic and on his insulin box it says 'insulin lispro (Rapid-
acting analogue)', unfortunately the instructions for Patients often require a mixture of preparations (e.g. both
administration have rubbed out. short and long acting) to ensure stable glycaemic control
throughout the day.
The nurse asks you how insulin lispro is usually
administered: Rapid-acting insulin analogues
• the rapid-acting human insulin analogues act faster and
A. Given just prior to meal
have a shorter duration of action than soluble insulin (see
B. Given just after meal
below)
C. Given 30 minutes before meal
• may be used as the bolus dose in 'basal-bolus' regimes
D. Given 30 minutes after meal
(rapid/short-acting 'bolus' insulin before meals with
E. Given only if blood sugar is found to be greater than
intermediate/long-acting 'basal' insulin once or twice
10mmol/L
daily)
• insulin aspart: NovoRapid
ANSWER:
• insulin lispro: Humalog
Given just prior to meal
Short-acting insulins
EXPLANATION:
Insulin lispro is a fast acting insulin analogue. Its levels peak • soluble insulin examples: Actrapid (human, pyr), Humulin
0-3 hours after injection and last 2-5 hours. It should S (human, prb)
therefore be give just prior to meals. • may be used as the bolus dose in 'basal-bolus' regimes
Q-100 What average blood glucose level for the past 2 months is
A 52-year-old woman who was diagnosed as having primary this most likely to represent?
atrophic hypothyroidism 12 months ago is reviewed
following recent thyroid function tests (TFTs): A. 9
B. 10
TSH 12.5 mU/l
C. 11
Free T4 14 pmol/l D. 15
E. There is no relation between HbA1c and average blood
She is currently taking 75mcg of levothyroxine once a day. glucose
How should these results be interpreted?
ANSWER:
A. Poor compliance with medication 15
B. Her thyroxine dose needs to be increased
C. Evidence of recent systemic steroid therapy EXPLANATION:
D. She is on the correct dose GLYCOSYLATED HAEMOGLOBIN
E. T4 to T3 conversion disorder
Glycosylated haemoglobin (HbA1c) is the most widely used
ANSWER: measure of long-term glycaemic control in diabetes mellitus.
Poor compliance with medication HbA1c is produced by the glycosylation of haemoglobin at a
rate proportional to the glucose concentration. The level of
EXPLANATION: HbA1c therefore is dependant on
The TSH level is high. This implies that over recent • red blood cell lifespan
days/weeks her body is thyroxine deficient. However, her • average blood glucose concentration
free T4 is within normal range. The most likely explanation is
that she started taking the thyroxine properly just before the A number of conditions can interfere with accurate HbA1c
blood test. This would correct the thyroxine level but the TSH interpretation:
takes longer to normalise.
Lower-than-expected levels of Higher-than-expected levels of
Please see Q-26 for Thyroid Function Tests HbA1c (due to reduced red blood HbA1c (due to increased red
cell lifespan) blood cell lifespan)
Sickle-cell anaemia Vitamin B12/folic acid deficiency
GP6D deficiency Iron-deficiency anaemia
Hereditary spherocytosis Splenectomy
Q-104
HbA1c is generally thought to reflect the blood glucose over A 52-year-old man has a set of fasting bloods as part of a
the previous '3 months' although there is some evidence it is work-up for hypertension. The fasting glucose comes back as
weighed more strongly to glucose levels of the past 2-4 6.5 mmol/l. The test is repeated and reported as 6.7 mmol/l.
weeks. NICE recommend 'HbA1c should be checked every 3-6 He says he feels constantly tired but denies any polyuria or
months until stable, then 6 monthly'. polydipsia. How should these results be interpreted?
The relationship between HbA1c and average blood glucose is A. Impaired fasting glycaemia
complex but has been studied by the Diabetes Control and B. Suggestive of diabetes mellitus but not diagnostic
Complications Trial (DCCT). A new internationally C. Diabetes mellitus
standardised method for reporting HbA1c has been developed D. Normal
by the International Federation of Clinical Chemistry (IFCC). E. Impaired glucose tolerance
This will report HbA1c in mmol per mol of haemoglobin
without glucose attached. ANSWER:
Impaired fasting glycaemia
HBA1c Average plasma glucose
(%) (mmol/l) IFCC-HbA1c (mmol/mol) EXPLANATION:
5 5.5
6 7.5 42 Please see Q-5 for Diabetes mellitus: management of type 2
7 9.5 53
8 11.5 64 Q-105
9 13.5 75 A 63-year-old woman presents to her GP for review. Her past
10 15.5 medical history includes hypothyroidism for which she
11 17.5 currently takes levothyroxine 100mcg. She is currently well
12 19.5 and has no symptoms of note. Her last thyroid function tests
(TFTs) 12 months ago on this dose were normal.
From the above we can see that average plasma glucose = (2 *
Free T4 18.5 pmol/l
HbA1c) - 4.5
TSH 0.1 mu/l
Q-103
A 62-year-old HGV driver is reviewed. He was diagnosed last What is the most appropriate action?
year with type 2 diabetes mellitus. Following weight loss and
metformin his HbA1c has decreased from 74 mmol/mol A. Make no changes to the current dose
(8.9%) to 68 mmol/mol (8.4%). What is the most suitable B. Increase dose to levothyroxine 150mcg od
next step in management? C. Decrease dose to levothyroxine 75mcg od
D. Decrease dose to levothyroxine 50mcg od
A. Add exenatide E. Add carbimazole 10mg od
B. Make no changes to management
C. Add gliclazide ANSWER:
D. Stop metformin for a period to ensure hypoglycaemic Decrease dose to levothyroxine 75mcg od
awareness is not lost
E. Add pioglitazone EXPLANATION:
The most recent TFTs show a suppressed TSH indicating over
ANSWER: replacement. Even though she is asymptomatic the dose
Add pioglitazone should be decreased to reduce the risk of osteoporosis and
atrial fibrillation. The BNF recommends adjusting the dose by
EXPLANATION: 25mcg in this age group.
Pioglitazone is the best option here as it would not put him
at risk of hypoglycaemia, which obviously could be Please see Q-10 for Hypothyroidism: Management
dangerous given his job. The NICE guidelines would also
support the use of a DPP-4 inhibitor (e.g. sitagliptin or Q-106
vildagliptin) in this situation. A 43-year-old woman is reviewed in clinic. She recently hard
a fasting plasma glucose measurement of 6.3 mmol/l as part
Please see Q-5 for Diabetes mellitus: management of type 2 of a work-up for lethargy. The GP registrar requested a
HbA1c to confirm the diagnosis of prediabetes. Which one of
the following conditions/situations is most likely to make
the result invalid?
A. Menorrhagia with a normal haemoglobin A. Continue to take the same dose
B. Idiopathic thrombocytopenic purpura B. Double hydrocortisone to 40mg mornings and 20mg
C. Haemolytic anaemia afternoon
D. Treated iron-deficiency anaemia C. Halve hydrocortisone to 10mg mornings and 5mg
E. Use of the combined oral contraceptive pill afternoon
D. Continue to take the same dose + prescribe a proton
ANSWER: pump inhibitor
Haemolytic anaemia E. Continue the same morning dose + stop the afternoon
dose
EXPLANATION:
Conditions where HbA1c may not be used for diagnosis: ANSWER:
• haemoglobinopathies Double hydrocortisone to 40mg mornings and 20mg
• haemolytic anaemia afternoon
• untreated iron deficiency anaemia
• suspected gestational diabetes EXPLANATION:
• children Patients on long-term steroids should have their doses
• HIV doubled during intercurrent illness
• chronic kidney disease
Please see Q-51 for Corticosteroids
Please see Q-5 for Diabetes mellitus: management of type 2
Q-109
Q-107 A 79-year-old man presents to his GP with a history of lower
You are discussing the results of a fasting blood sugar with a back pain and right hip pain. Blood tests reveal the
patient. It was done after the patient was found to be following:
hypertensive:
Calcium 2.20 mmol/l
Complications
• deafness (cranial nerve entrapment)
• bone sarcoma (1% if affected for > 10 years)
• fractures
Isotope bone scan from a patient with Paget's disease showing a typical
• skull thickening distribution in the spine, asymmetrical pelvic disease and proximal long bones.
• high-output cardiac failure
*usually normal in this condition but hypercalcaemia may
occur with prolonged immobilisation
Q-110
A 45-year-old woman with Graves' disease comes for review.
She has recently been diagnosed with thyroid eye disease
and is being considered for radiotherapy. Over the past three
days her right eye has become red and painful. On
examination there is proptosis and erythema of the right
eye. Visual acuity is 6/9 in both eyes. What complication is
she most likely to have developed?
A. Exposure keratopathy
B. Optic neuropathy
C. Carbimazole-related neutropaenia
The radiograph demonstrates marked thickening of the calvarium. There are also D. Central retinal vein occlusion
ill-defined sclerotic and lucent areas throughout. These features are consistent E. Sjögren’s Syndrome
with Paget's disease.
ANSWER:
Exposure keratopathy
EXPLANATION:
Please see Q-1 for Thyroid Eye Disease
Q-111
A 58-year-old man comes for review in the diabetes clinic.
He was diagnosed as having type 2 diabetes mellitus (T2DM)
around 10 years ago and currently only takes gliclazide and
atorvastatin. Three years ago he was successfully treated for
bladder cancer. A recent trial of metformin was unsuccessful
due to gastrointestinal side-effects. He works as an
accountant, is a non-smoker and his BMI is 31 kg/m². His
annual bloods show the following:
Pelvic x-ray from an elderly man with Paget's disease. There is a smooth cortical
expansion of the left hemipelvic bones with diffuse increased bone density and
coarsening of trabeculae.
Na+ 138 mmol/l A. Bicuspid aortic valve
B. Previous malignant melanoma
K+ 4.1 mmol/l
C. Epilepsy
Urea 4.3 mmol/l
D. Type 2 diabetes
Creatinine 104 µmol/l E. Acne vulgaris
HbA1c 62 mmol/mol (7.8%)
ANSWER:
What is the most appropriate next step in management?
Epilepsy
A. Add pioglitazone
EXPLANATION:
B. Add exenatide
The AKT summary report for the October 2014 exam sitting
C. Add acarbose
noted that candidates had trouble with questions regarding
D. Add repaglinide
the management of obesity. Please see the link below for the
E. Add sitagliptin
full NICE guidance on obesity, published in 2014.
ANSWER:
Orlistat is a gastrointestinal lipase inhibitor which is used to
Add sitagliptin
treat obesity it reduces the absorption of fat from the gut.
This commonly causes loose stool/diarrhoea unless a low-fat
EXPLANATION:
diet is strictly adhered to. Therefore, it is important to
Pioglitazone is contraindicated by his history of bladder
consider whether orlistat is suitable when the patient takes
cancer and may contribute to his obesity. A DPP-4 inhibitor
critical medications such as antiepileptics or the
such as sitagliptin is therefore the best option.
contraceptive pill, where increased transit time through the
gut caused by orlistat could reduce absorption and therefore
Exenatide generally causes weight loss and is therefore
efficacy of critical medications. Antiepileptics + orlistat is
useful in obese diabetics but he does not meet the NICE body
listed as a red interaction in the British National Formulary
mass index criteria of 35 kg/m².
(BNF).
Please see Q-5 for Diabetes mellitus: management of type 2
Please see Q-21 for Obesity: Therapeutic Options
Q-112
Q-114
A 56-year-old man is reviewed in the Cardiology outpatient
You are called on a home visit to see a 66-year-old male
clinic following a myocardial infarction one year previously.
patient with a past medical history of type 1 diabetes
During his admission he was found to be hypertensive and
mellitus, ischaemic heart disease, ischaemic stroke,
diabetic. He complains that he has put on 5kg in weight in
peripheral neuropathy and depression. When you arrive, the
the past 6 months. Which of his medications may be
patient's wife directs you upstairs to the patient's bedroom,
contributing to his weight gain?
however, when you arrive you find him collapsed on the
floor. You ask the wife to phone 999 and start your ABCDE
A. Metformin
assessment. The patient has a patent airway, has a
B. Losartan
respiratory rate of 24/min and pulse of 124/min. He is
C. Clopidogrel
unresponsive to pain when applying supraorbital pressure.
D. Gliclazide
You do a blood glucose and find it is 1.1. What is the best
E. Simvastatin
initial management?
ANSWER:
A. Give 20g glucose by mouth
Gliclazide
B. Give 1mg glucagon intramuscularly
C. Give the patient a sandwich
EXPLANATION:
D. Give 50mL of glucose intravenous infusion 20%
Please see Q-27 for Sulfonylureas
E. Give water with 2 teaspoons of dissolved sugar (equals
approximately 10mg of glucose)
Q-113
A 23-year-old patient makes an appointment to see her GP
ANSWER:
to request a prescription for orlistat. Which co-morbid
Give 1 mg glucagon intramuscularly
condition is most likely to be a contraindication to the
prescription of this medication?
EXPLANATION:
Option 2 is the correct answer. This patient has a low level of
consciousness and is not responding to pain, therefore trying
to give glucose, drinks or carbohydrates by mouth isn't the
best option. In this scenario, glucagon can be given by EXPLANATION:
injection. Giving a glucose infusion would more likely be Prediabetes is defined by a HbA1c of 42-47 mmol/mol (6.0-
given in hospital than in the community and is not as good 6.4%)
an initial treatment as giving glucagon intramuscularly.
Please see Q-2 for Prediabetes and Impaired Glucose
'Serious medical emergencies are relatively uncommon in Regulation
general practice. However, when events do occur, immediate
intervention with administration of emergency drugs may be Q-117
required. Candidates should be familiar with the A 45-year-old man is reviewed in the diabetes clinic. The
management, including drug treatment, of patients following results are obtained:
presenting with life-threatening conditions.'
Urinalysis NAD
October 2016 AKT Feedback Report HbA1c 69 mmol/mol
Gliclazide is added to the metformin he already takes. What
Please see Q-59 for Insulin Therapy: Side Effects is the minimum time period after which the HbA1c should be
repeated?
Q-115
A 43 year old woman with Addison's disease presents to the A. 6 months
surgery with a chest infection requiring antibiotics. What B. 1 month
should you do regarding her steroid replacement? C. 2 weeks
D. 3 months
A. Half hydrocortisone dose, same fludrocortisone dose E. 4 months
B. Double hydrocortisone dose, double fludrocortisone ANSWER:
dose 3 months
C. Hold hydrocortisone as patient has infection
D. Double hydrocortisone dose, same fludrocortisone dose EXPLANATION:
E. Same hydrocortisone dose, double fludrocortisone dose NICE advise monitoring HbA1c '2-6 monthly (according to
individual needs) until stable on unchanging therapy'.
ANSWER:
Double hydrocortisone dose, same fludrocortisone dose Please see Q-102 for Glycosylated Haemoglobin
Q-118
EXPLANATION: A 42-year-old man presents to his GP feeling generally
Patients with Addison's have little or no endogenous steroid unwell. For the past three months he has been experiencing
production. During illness the body usually increases cortisol daily frontal headaches which have not been helped by
production as a stress response. Therefore hydrocortisone regular paracetamol. He has also noticed some unusual
doses are typically doubled while such patients are unwell. symptoms such as his wedding ring no longer fitting, his shoe
Fludrocortisone is less important and the dose is typically size apparently increasing and a small amount of milky
kept the same as normal. discharge from both nipples. On examination his blood
pressure is 168/96 mmHg. What is the most likely diagnosis?
Please see Q-43 for Addison’s Disease: Management
A. Phaeochromocytoma
Q-116 B. Cushing's syndrome
You are reviewing the blood results of a 67-year-old man C. Diabetes insipidus
who has recently been diagnosed as having hypertension. A D. Macroprolactinoma
HbA1c level was requested as part of the routine work-up. E. Acromegaly
Which one of the following HbA1c ranges is most consistent
with a diagnosis of prediabetes? ANSWER:
Acromegaly
A. 31-36 mmol/mol (5.0-5.4%) EXPLANATION:
B. 37-41 mmol/mol (5.5-5.9%) ACROMEGALY: FEATURES
C. 42-47 mmol/mol (6.0-6.4%)
D. 48-52 mmol/mol (6.5-6.9%) In acromegaly there is excess growth hormone secondary to a
E. 53-58 mmol/mol (7.0-7.5%) pituitary adenoma in over 95% of cases. A minority of cases
are caused by ectopic GHRH or GH production by tumours e.g.
ANSWER: pancreatic
42-47 mmol/mol (6.0-6.4%)
Features Q-121
• coarse facial appearance, spade-like hands, increase in A 52-year-old man who has recently being diagnosed with
shoe size hypertension is reviewed. He describes having some recent
• large tongue, prognathism, interdental spaces polydipsia and polyuria. As part of his initial work-up a
• excessive sweating and oily skin HbA1c was requested. What is the lowest IFCC-HbA1c (not
• features of pituitary tumour: hypopituitarism, headaches, DCCT-HbA1c) value that would indicate a diagnosis of
bitemporal hemianopia diabetes mellitus?
• raised prolactin in 1/3 of cases → galactorrhoea
• 6% of patients have MEN-1 Answer: ____ mmol/mol
Complications ANSWER:
• hypertension 48 mmol/mol
• diabetes (>10%)
• cardiomyopathy EXPLANATION:
Please see Q-5 for Diabetes mellitus: management of type 2
• colorectal cancer
Q-122
Q-119
A 39-year-old woman who has a history of type 1 diabetes
One of your patients is diagnosed with having the metabolic
mellitus phones for advice as she is worried about her blood
syndrome. Which one of the following is associated with this
sugar level. What is the target blood sugar level before meals
condition?
at other times of the day (excluding first thing in the
morning)?
A. Endometriosis
B. Hypothyroidism
A. 5-9 mmol/l
C. Asymptomatic rise in amylase levels
B. 4-7 mmol/l
D. Elevated albumin levels
C. 4-6 mmol/l
E. Raised uric acid levels
D. 6-8 mmol/l
E. 5-7 mmol/l
ANSWER:
Raised uric acid levels
ANSWER:
4-7 mmol/l
EXPLANATION:
Please see Q-35 for Metabolic Syndrome
EXPLANATION:
Q-120 In type 1 diabetics, blood glucose targets:
A 54-year-old man has a routine medical for work. He is • 5-7 mmol/l on waking and
asymptomatic and clinical examination is unremarkable. • 4-7 mmol/l before meals at other times of the day
Which of the following results establishes a diagnosis of
impaired fasting glucose? Please see Q-17 for Diabetes Mellitus: Management of Type
1
A. Fasting glucose 7.1 mmol/L on one occasion
B. Fasting glucose 6.8 mmol/L on two occasions Q-123
C. Glycosuria ++ Which one of the following drugs is least likely to cause
D. 75g oral glucose tolerance test 2 hour value of 8.4 gynaecomastia?
mmol/L
E. HbA1c of 6.7% A. Spironolactone
B. Sodium valproate
ANSWER: C. Digoxin
Fasting glucose 6.8 mmol/L on two occasions D. Cimetidine
E. Anabolic steroids
EXPLANATION:
A 75g oral glucose tolerance test 2 hour value of 8.4 mmol/L ANSWER:
would imply impaired glucose tolerance rather than Sodium valproate
impaired fasting glucose
EXPLANATION:
Please see Q-5 for Diabetes mellitus: management of type 2 Whilst sodium valproate may rarely causes gynaecomastia it
is much more common after taking the other listed drugs.
Please see Q-60 for Kallman’s Syndrome Please see Q-21 for Obesity: Therapeutic Options
Q-125 Q-127
A patient is diagnosed with type 2 diabetes mellitus and A 51-year-old woman who is known to have poorly
started on metformin monotherapy. Following NICE controlled type 1 diabetes mellitus is reviewed. Her main
guidelines, what target should be set for the HbA1c? presenting complaint is bloating and vomiting after eating.
She also notes that her blood glucose readings have become
A. Agree target with patient but generally aim for 58 more erratic recently. Which one of the following
mmol/mol medications is most likely to be beneficial?
B. Agree target with patient but generally aim for 42
mmol/mol A. Helicobacter pylori eradication therapy
C. As low as possible B. Lansoprazole
D. Agree target with patient but generally aim for 48 C. Amitriptyline
mmol/mol D. Metoclopramide
E. 54 mmol/mol E. Cyclizine
ANSWER: ANSWER:
Agree target with patient but generally aim for 48 mmol/mol Metoclopramide
EXPLANATION: EXPLANATION:
As this patient is on metformin monotherapy the target
should be 48 mmol/mol (6.5%). If he was on dual therapy a Please see Q-40 for Diabetic Neuropathy
target of 53 mmol/mol (7.0%) would be recommend by NICE.
Please see Q-5 for Diabetes mellitus: management of type 2
Q-128 Q-130
A 50-year-old bank clerk is seen in the diabetes clinic. He has You review a 68-year-old man who has chronic obstructive
type 2 diabetes mellitus which is currently treated with pulmonary disease (COPD). Each year he typically has around
metformin. Unfortunately his glycaemic control is 7-8 courses of oral prednisolone to treat infective
suboptimal. He is intolerant of sulfonylureas and exacerbations of his COPD. Which one of the following
thiazolidinediones and it is decided to add exenatide. What adverse effects is linked to long-term steroid use?
is the most appropriate action with respect of the DVLA?
A. Osteomalacia
A. Inform DVLA but can continue to drive B. Enophthalmos
B. Inform DVLA, must check blood sugars before journey C. Leucopaenia
and at least every 2 hours D. Avascular necrosis
C. No need to inform DVLA E. Constipation
D. Inform DVLA, cannot drive until 4 weeks have passed
without hypoglycaemic episodes ANSWER:
E. Inform DVLA, cannot drive until 3 months have passed Avascular necrosis
without hypoglycaemic episodes
EXPLANATION:
ANSWER: Long-term corticosteroid use is linked to osteopaenia and
No need to inform DVLA osteoporosis, rather than osteomalacia.
ANSWER: EXPLANATION:
Refer to the local diabetic foot centre This lady has symptomatic hypothyroidism and needs
thyroxine replacement. The BNF guidelines suggest a
EXPLANATION: starting dose for patients < 50 years of 50-100 mcg od.
Diabetic patients who have any foot problems other than Studies have also shown that an initial treatment dose of
simple calluses should be followed up regularly by the local 1.6mcg/kg/day is suitable for younger patients without
diabetic foot centre heart disease. The answer is therefore consistent with both
the BNF advice and relevant clinical trials.
Please see Q-67 for Diabetic Foot Disease
Please see Q-10 for Hypothyroidism: Management
Q-132
A 23-year-old woman presents with sweating and tremor.
Her thyroid function tests are as follows:
Q-133
TSH <0.05 mU/l
A 54-year-old man is reviewed shortly after being diagnosed
with hypertension. as part of his work-up he had a series of
Free T4 25 pmol/l
blood tests to screen for other risk factors:
What is the most common cause this presentation? Na+ 142 mmol/l
K+ 3.9 mmol/l
A. Hashimoto's thyroiditis
Urea 6.2 mmol/l
B. Graves' disease
C. Toxic nodular goitre Creatinine 91 µmol/l
D. De Quervain's thyroiditis
E. Toxic adenoma Fasting glucose 7.7 mmol/l
Investigation Q-134
• TSH down, T4 and T3 up A 79-year-old woman presents after having a fall. During the
• thyroid autoantibodies consultation she complains of feeling cold all the time.
• other investigations are not routinely done but includes Thyroid function tests (TFTs) are therefore ordered:
isotope scanning
Free T4 7.1 pmol/l
ANSWER:
Start levothyroxine 25mcg od
Venn diagram showing how different causes of thyroid dysfunction may manifest.
Note how many causes of hypothyroidism may have an initial thyrotoxic phase.
EXPLANATION: A. < 125 - 75 mmHg
This lady has hypothyroidism as evidence by the low free T4 B. < 130 - 75 mmHg
and raised TSH. Given her age levothyroxine should be C. < 130 - 80 mmHg
introduced slowly starting with a dose of 25mcg od. D. < 140 - 80 mmHg
E. < 140 - 85 mmHg
Please see Q-10 for Hypothyroidism: Management
ANSWER:
Q-135 < 140 - 80 mmHg
A 46-year-old man with suspected diabetes mellitus has an
oral glucose tolerance test, following the standard WHO EXPLANATION:
protocol. The following results are obtained: Type 2 diabetes blood pressure target
• no organ damage: < 140 / 80
Time (hours) Blood glucose (mmol/l) • end-organ damage: < 130 / 80
0 5.7 The April 2010 AKT feedback report stated: 'Items concerning
routine management of type 2 diabetes caused some
2 7.6
problem, especially with regard to the wider management of
cardiovascular risks.'
How should these results be interpreted?
Please see Q-5 for Diabetes mellitus: management of type 2
A. Normal
B. Impaired fasting glucose and impaired glucose tolerance Q-138
C. Diabetes mellitus At his 3-month follow-up after being prescribed orlistat for
D. Impaired glucose tolerance the management of obesity (BMI 33), a patient says he has
E. Impaired fasting glucose been unable to take the medication as prescribed due to the
side effects he has experienced. He is aged 53, with a past
ANSWER: medical history of psoriasis, hyperlipidaemia and
Normal hypertension. Which alternative treatment does NICE
recommend may be suitable for this patient?
EXPLANATION:
Both the fasting and two-hour glucose are within normal A. Silbutramine
limits. B. Acarbose
C. Exenatide
Please see Q-6 for Diabetes Mellitus (Type 2): Diagnosis D. Bariatric surgery
E. None of the above
Q-136
Which one of the following drugs is not associated with ANSWER:
galactorrhoea? None of the above
A. Metoclopramide EXPLANATION:
B. Bromocriptine The AKT summary report for the October 2014 exam sitting
C. Chlorpromazine noted that there seemed to be lack of knowledge around
D. Haloperidol current guidance on the management of obesity. Please see
E. Domperidone the link below for the full NICE guidance on obesity,
published in 2014.
ANSWER:
Bromocriptine None of the first three options listed above are currently
advised for the treatment of obesity (though exenatide may
EXPLANATION: have the side effect of promoting weight loss when
Bromocriptine is a treatment for galactorrhoea, rather than prescribed in type 2 diabetes). Silbutramine is no longer
a cause prescribable in the UK (see notes below). Nor does he meet
the current eligibility criteria for bariatric surgery, which
Please see Q-66 for Prolactin and Galactorrhoea were broadened somewhat in the 2014 guidelines.
Q-139 Q-141
You are reviewing a 31-year-old woman who has type 1 A 51-year-old woman is reviewed in the diabetes clinic. She
diabetes mellitus. Her control is currently good and she is was diagnosed with type 2 diabetes mellitus 12 months ago
well with no intercurrent illnesses. How often if it and still has poor glycaemic control (63 mmol/mol). She has
recommended that she monitors her blood glucose? recently had to stop taking gliclazide due to repeated
episodes of hypoglycaemia and is only taking maximum dose
A. At least 4 times a day, including before each meal and metformin. Her BMI is 26 kg/m^2. What is the most
before bed appropriate next step in management?
B. At least 6 times a day, including before each meal and
before bed A. Add either pioglitazone, a DPP-4 inhibitor or a SGLT-2
C. At least 8 times a day, including before each meal and inhibitor
before bed B. Refer her for a laparoscopic gastric band
D. On waking and after lunch and the evening meal C. Refer her for insulin therapy
E. On waking, before leaving the house and after lunch and D. Add either a thiazolidinedione or exenatide
the evening meal E. Add either a DPP-4 inhibitor or exenatide
ANSWER: ANSWER:
At least 4 times a day, including before each meal and before Add either pioglitazone, a DPP-4 inhibitor or a SGLT-2 inhibitor
bed
EXPLANATION:
EXPLANATION: Please see Q-5 for Diabetes mellitus: management of type 2
In type 1 diabetics, recommend monitoring blood glucose at
least 4 times a day, including before each meal and before Q-142
bed A 68-year-old woman is found to have the following blood
tests:
Please see Q-17 for Diabetes Mellitus: Management Of Type
1 TSH 0.05 mu/l
Q-148
What is the most appropriate management?
Which one of the following is not part of the diagnostic
criteria for the metabolic syndrome?
A. Carbimazole 20 mg daily 30 minutes before breakfast,
caffeine and other medication(s)
A. High triglycerides
B. Carbimazole 20 mg daily with food
B. Low HDL
C. Levothyroxine 75 mcg daily 30 minutes before breakfast,
C. High LDL
caffeine and other medication(s)
D. Central obesity
D. Levothyroxine 75mcg daily with food
E. Hypertension
E. Refer to endocrine clinic
ANSWER:
ANSWER:
High LDL
Levothyroxine 75 mcg daily 30 minutes before breakfast,
caffeine and other medication(s)
EXPLANATION:
High LDL levels are not part of the World Health
EXPLANATION:
Organization or International Diabetes Federation diagnostic
The TFTs show hypothyroidism and the treatment is
criteria
levothyroxine. Carbimazole is used to treat hyperthyroidism
and would not be appropriate in this case. Levothyroxine
Please see Q-35 for Metabolic Syndrome
must be taken 30 minutes before food as it's absorption may
be affected by food, caffeine or other medications.
Q-149
You receive a letter from an endrocinology consultant
Please see Q-10 for Hypothyroidism: Management
following a referral that you made for a 23 year old lady who
has been newly diagnosed with hyperthyroidism. The
Q-151
consultant requests that you start the patient on
A 60-year-old man is found to have a QRISK2 score of 14%
carbimazole 15mg daily with a repeat thyroid function test
after having his cholesterol levels checked. After discussing
(TFT) in 4 weeks. What is the most important advice to give
the pros and cons he elects to start atorvastatin 20mg on.
the patient?
Total cholesterol 5.6 mmol/l
A. Do not stop taking the medication suddenly
B. Attend for urgent medical review if develops any HDL cholesterol 1.0 mmol/l
symptoms of infection e.g. sore throat or fever LDL cholesterol 3.4 mmol/l
C. Must have monthly liver function tests (LFT) Triglyceride 1.7 mmol/l
D. Must have monthly full blood count (FBC)
E. Must have monthly urea & electrolytes (UE)
At what point should he have a repeat cholesterol test done
to test the effectiveness of the statin?
ANSWER:
Attend for urgent medical review if develops any symptoms of
infection e.g. sore throat or fever A. 6 weeks
B. 12 weeks
EXPLANATION: C. 6 months
A rare but serious side effect of carbimazole is D. 12 months
agranulocytosis so patients must be counselled regarding E. Checking cholesterol levels is not needed in primary
prevention
this. If the patient develops any symptoms of an infection,
particularly sore throat or fever then must seek urgent
medical review and a FBC must be performed to check the ANSWER:
12 weeks
neutrophil count.
EXPLANATION:
A lipid profile and liver function tests should be performed 3 Measuring lipid levels
months after starting a statin When measuring lipids both the total cholesterol and HDL
NICE recommend the following: should be checking to provide the most accurate risk of CVD.
A full lipid profile should also be checked (i.e. including
What follow up is recommended after initiation of statin triglycerides) before starting a statin. The samples does not
therapy for primary prevention? need to be fasting.
Measure total cholesterol, high-density lipoprotein (HDL)
cholesterol, and non-HDL cholesterol (total cholesterol minus In the vast majority of patient the cholesterol measurements
HDL cholesterol) levels after 3 months of atorvastatin will be fed into the QRISK2 tool. If however the patient's
treatment. The aim of treatment is to achieve a greater than cholesterol is very high we should consider familial
40% reduction in baseline non-HDL cholesterol levels. hyperlipidaemia. NICE recommend the following that we
should consider the possibility of familial
HYPERLIPIDAEMIA: MANAGEMENT hypercholesterolaemia and investigate further if the total
In 2014 NICE updated their guidelines on lipid modification. cholesterol concentration is > 7.5 mmol/l and there is a family
This proved highly controversial as it meant that we should be history of premature coronary heart disease. They also
recommending statins to a significant proportion of the recommend referring people with a total cholesterol > 9.0
population over the age of 60 years. Anyway, the key points of mmol/l or a non-HDL cholesterol (i.e. LDL) of > 7.5 mmol/l
the new guidelines are summarised below. even in the absence of a first-degree family history of
premature coronary heart disease.
EXPLANATION: Q-158
A 70-year-old man is reviewed. For the past two weeks his
Please see Q-151 for hyperlipidaemia: management family report that he has been generally unwell, 'not himself'
and lethargic. His past history includes ischaemic heart
Q-157 disease (myocardial infarction 8 years ago), chronic kidney
You are reviewing a 35-year-old man who has just been disease (stage 4) and depression (for which he takes
diagnosed with familial hypercholesterolaemia. He is imipramine). Physical examination is unremarkable with no
concerned that his children (the youngest of whom is 7- new findings. You obtain an ECG:
years-old) may have inherited the condition and asks about
screening. His wife has been tested and is not affected. What
is the most appropriate response with respect to screening?
EXPLANATION: ANSWER:
If we feed his age, gender and smoking history into QRISK2 1 in 500 people
this gives a 10-year-risk of cardiovascular disease (CVD) of
13.9%. He is therefore an appropriate person to have a EXPLANATION:
'formal' assessment of CVD risk using a lipid profile to further Please see Q-157 for Familial Hypercholesterolaemia
inform the QRISK2 score.
Q-165
Please see Q-151 for Hyperlipidaemia: Management A 54-year-old man is reviewed in clinic. He has recently
started atorvastatin 20mg after it was found that his QRISK2
Q-163 score was 16%. He asks you about any changes he should
A 69-year-old female with a history of multiple myeloma make to his diet. Which one of the following is a part of
presents with confusion. Blood tests are taken and the NICE's current advice on a cardioprotective diet?
following results are obtained:
A. Eat at least 7 portions of fruit and vegetables a day
Adjusted calcium 3.1 mmol/l B. Increase the amount of fructose in diet
C. Eat at least 4 to 5 portions of unsalted nuts, seeds and
What is the most appropriate initial management? legumes per week
D. Saturated fats should be between 7-10% of total energy
A. Oral alendronate + prednisolone intake
B. Oral alendronate E. Monounsaturated spreads are preferred to those
C. Oral prednisolone derived from olive oil
D. Admit for IV pamidronate
E. Admit for IV normal saline ANSWER:
Eat at least 4 to 5 portions of unsalted nuts, seeds and
ANSWER: legumes per week
Admit for IV normal saline
EXPLANATION:
EXPLANATION:
IV fluid therapy is the first-line management in patients with Please see Q-151 for Hyperlipidaemia: Management
hypercalcaemia
Q-166
HYPERCALCAEMIA: MANAGEMENT A 67-year-old man is discharged after having a percutaneous
The initial management of hypercalcaemia is rehydration with coronary intervention following an acute coronary syndrome
normal saline, typically 3-4 litres/day. Following rehydration (ACS). He had no past medical of note prior to the ACS.
bisphosphonates may be used. They typically take 2-3 days to Which type of lipid modification therapy should he have
work with maximal effect being seen at 7 days been started on during the admission?
EXPLANATION: EXPLANATION:
Patients with established CVD should take atorvastatin 80mg Cod liver oil provides around 1,300 IU per 15 ml serving
on
VITAMIN DEFICIENCY
Please see Q-151 for Hyperlipidaemia: Management The table below summarises vitamin deficiency states
For each one of the following scenarios select the vitamin HYPONATRAEMIA
which may cause these features if deficient:
Hyponatraemia may be caused by water excess or sodium
Q-173 depletion. Causes of pseudohyponatraemia include
Bleeding gums hyperlipidaemia (increase in serum volume) or a taking blood
from a drip arm. Urinary sodium and osmolarity levels aid
ANSWER: making a diagnosis
Vitamin C
Urinary sodium > 20 mmol/l
Q-174
Diarrhoea, confusion and eczematous skin Sodium depletion, renal loss (patient often hypovolaemic)
• diuretics
ANSWER: • Addison's
Niacin • diuretic stage of renal failure
EXPLANATION:
Na+ 129 mmol/l
Please see Q-158 for Hyperkalaemia
K+ 3.8 mmol/l
EXPLANATION: Q-183
A sodium result of 118 mmol/L falls into the category of Neural tube defects
severe hyponatraemia (less than 125 mmol/L). Emergency
admission is indicated because hyponatraemia is potentially ANSWER:
life threatening, particularly when it is severe and/or of Folic acid
acute onset (over a period of less than 48 hours). This is due
to swelling of brain cells when water moves from the Q-184
extracellular to the intracellular compartment because of Haemorrhagic disease of the newborn
differences in the osmolality between brain and plasma.
Cerebral oedema and raised intracranial pressure can lead to ANSWER:
seizures, coma or cardio-respiratory arrest. Vitamin K
Hyponatraemia can be classified by biochemical severity and EXPLANATION Q-182 THROUGH 184:
rate of onset: Please see Q-168 for Vitamin Deficiency
• Biochemical severity
• mild hyponatraemia = serum sodium 130-135 mmol/L Q-185
• moderate hyponatraemia = serum sodium 125-129 Fred is a 55-year-old man who has been diagnosed with lung
mmol/L cancer. He is currently on chemotherapy. He presents to you
• severe hyponatraemia = serum sodium less than 125 with a 2-week history of constipation and nausea. He has
mmol/L been finding it difficult to cope and becoming tearful and
depressed. He has also noticed that his arms and legs have
Rate of onset been increasingly aching. You arrange for a routine set of
• acute = hyponatraemia duration for less than 48 hours bloods:
• chronic = hyponatraemia duration for 48 hours or more.
Hb 109 g/l
Creatinine 80 µmol/l
Corrected Ca2+ 3.2 mmol/l
Cardiovascular
Albumin 30 g/l
• Hypertension.
ALP 99 u/l
• Shortened QT interval on electrocardiogram (ECG).
Phosphate 1.06 mmol/l • Cardiac arrhythmias (rare).
ANSWER: Which one of the following would not explain this result?
Admit immediately
A. Delay in transport to the laboratory
EXPLANATION: B. Losartan therapy
Fred is developing symptomatic metastatic hypercalcaemia. C. Addison's disease
D. Acute renal failure
The NICE Clinical Knowledge Summaries (CKS) state that if a E. Conn's syndrome
patient has symptomatic hypercalcaemia or moderate to
severe hypercalcaemia (adjusted serum calcium ANSWER:
concentration greater than 3.0 mmol/L) then the patient Conn’s syndrome
should be admitted immediately for intravenous fluids and
bisphosphonate therapy. EXPLANATION:
Conn's syndrome is associated with hypokalaemia.
Remember the clinical symptoms of hypercalcaemia can be
very non-specific. The following list is provided in the NICE Please see Q-158 for Hyperkalaemia
CKS:
Q-187
Skeletal A 43-year-old man requests a 'medical' as he is concerned
• Bone pain. about his risk of heart disease. His father died at the age of
• Fractures associated with underlying bone disorders 45-years following a myocardial infarction. His lipid profile is
(fragility fractures in hyperparathyroidism or as follows:
pathological fractures in malignancy).
HDL 1.4 mmol/l
ANSWER: ANSWER:
Switch to atorvastatin 80 mg on Atorvastatin 20 mg on
EXPLANATION: EXPLANATION:
NICE recommend the following when considering the use of
Please see Q-151 for hyperlipidaemia: management statins in patients with type 2 diabetes mellitus:
ANSWER: EXPLANATION:
Stop atorvastatin before trying to conceive As 1/3 of patients have infrequent relapses and 1/3 of
patients have frequent relapses a majority (2/3) will have
EXPLANATION: later recurrent episodes. It is important however to stress to
Statins should be discontinued in women 3 months before patients that generally speaking the longer term prognosis in
conception due to the risk of congenital defects minimal change glomerulonephritis is good.
K+ 2.6 mmol/l
CKD
stage GFR range Bicarbonate 33 mmol/l
1 Greater than 90 ml/min, with some sign of kidney damage on Urea 4.2 mmol/l
other tests (if all the kidney tests* are normal, there is no CKD)
Creatinine 91 µmol/l
2 60-90 ml/min with some sign of kidney damage (if kidney tests*
are normal, there is no CKD)
3a 45-59 ml/min, a moderate reduction in kidney function Which one of the following conditions is most likely to be
3b 30-44 ml/min, a moderate reduction in kidney function responsible?
4 15-29 ml/min, a severe reduction in kidney function
5 Less than 15 ml/min, established kidney failure - dialysis or a
A. Cushing's syndrome
kidney transplant may be needed B. Conn's syndrome
C. 11-beta hydroxylase deficiency
*i.e. normal U&Es and no proteinuria D. Bartter's syndrome
E. Liddle's syndrome
Q-197
A 65-year-old man with a history of hypertension is ANSWER:
reviewed. As part of routine blood tests to monitor his renal Bartter's syndrome
function whilst taking ramipril the following blood tests are
received: EXPLANATION:
Bartter's syndrome is associated with normotension
Na+ 140 mmol/l Bartter's syndrome is an inherited cause (usually autosomal
K +
4.8 mmol/l
recessive) of severe hypokalaemia due to defective chloride
absorption at the Na+ K+ 2Cl- cotransporter in the ascending
Urea 6.2 mmol/l
loop of Henle
Creatinine 102 µmol/l
eGFR 68 ml/min
HYPOKALAEMIA AND HYPERTENSION
For exams it is useful to be able to classify the causes of
hypokalaemia in to those associated with hypertension, and
A urine dipstick is subsequently performed which is normal
those which are not
and a renal ultrasound sound shows normal sized kidneys
with no abnormality detected. What stage of chronic kidney
disease does this patient have?
Hypokalaemia with hypertension Others
• Cushing's syndrome • congenital
• Conn's syndrome (primary hyperaldosteronism) • neoplasia: carcinoma, lymphoma, leukaemia, myeloma
• Liddle's syndrome • infection: bacterial endocarditis, hepatitis B, malaria
• 11-beta hydroxylase deficiency*
Q-200
**type 4 renal tubular acidosis is associated with A 47-year-old man comes to see you as the nurse found that
hyperkalaemia his blood pressure was raised. He suffers from chronic
kidney disease and his estimated glomerular filtration rate 6
Q-199 weeks ago was 53ml/min. Their albumin:creatinine ratio was
A 54-year-old female with rheumatoid arthritis is noted to 35mg/mmol. He does not complain of any chest pains or
have proteinuria on annual review. Which one of the shortness of breath.
following drugs is most associated with the development of
proteinuria? On examination, his blood pressure is 172/94 mmHg.
Fundoscopy is normal.
A. Ciclosporin
B. Gold What is the best treatment option?
C. Methotrexate
D. Infliximab A. Amlodipine
E. Sulfasalazine B. Bisoprolol
C. Doxazosin
ANSWER: D. Indapamide
Gold E. Ramipril
EXPLANATION: ANSWER:
NEPHROTIC SYNDROME: CAUSES Ramipril
For each one of the following scenarios please select the K+ 4.5 mmol/l
most likely diagnosis:
Creatinine 116 µmol/l
EXPLANATION: ANSWER:
Wilms' nephroblastoma is one of the most common childhood No action
malignancies. It typically presents in children under 5 years of
age, with a median age of 3 years old. EXPLANATION:
The small change in both the creatinine and eGFR are
Features acceptable and below the threshold where ACE inhibitors
• abdominal mass (most common presenting feature) should be stopped
Please see Q-200 for Chronic Kidney Disease: Hypertension
Q-211 Q-212
A 33-year-old pregnant woman presents for advice. She is Which one of the following factors is most likely to invalidate
known to have polycystic kidney disease but is currently the use of the Modification of Diet in Renal Disease (MDRD)
well. Her father also has polycystic kidneys and is on dialysis. equation to calculate a patient’s eGFR?
What is the chance her child will also have the disease?
A. Diuretic use
A. 50% if male B. Pregnancy
B. 50% C. Type 2 diabetes mellitus
C. 25% D. Blood pressure of 180/110 mmHg
D. 0% E. Female gender
E. 100%
ANSWER:
ANSWER: Pregnancy
50%
EXPLANATION:
EXPLANATION: GFR tends to increase during pregnancy although the eGFR
Polycystic kidney disease is usually inherited in an autosomal may not reflect this.
dominant fashion and hence 50% of her children will be
affected, regardless of gender. The autosomal recessive form Please see Q-196 for Chronic Kidney Disease: eGFR and
is rare and usually causes death in childhood. Classification
ADPKD Q-213
A 34-year-old man presents to an emergency surgery with
Autosomal dominant polycystic kidney disease (ADPKD) is the abdominal pain. This started earlier on in the day and is
most common inherited cause of kidney disease, affecting 1 in getting progressively worse. The pain is located on his left
1,000 Caucasians. Two disease loci have been identified, PKD1 flank and radiates down into his groin. He has had no similar
and PKD2, which code for polycystin-1 and polycystin-2 pain previously and is normally fit and well. Examination
respectively reveals a man who is flushed and sweaty but is otherwise
unremarkable. What is the most suitable initial
ADPKD type 1 ADPKD type 2 management?
85% of cases 15% of cases
Chromosome 16 Chromosome 4 A. Oral ciprofloxacin
Presents with renal failure earlier B. IM diclofenac 75 mg
C. Oral co-amoxiclav and metronidazole
The screening investigation for relatives is abdominal D. IM morphine 5 mg
ultrasound: E. IM diclofenac 75 mg + start bendroflumethiazide to
prevent further episodes
Ultrasound diagnostic criteria (in patients with positive family
history) ANSWER:
• two cysts, unilateral or bilateral, if aged < 30 years IM diclofenac 75 mg
• two cysts in both kidneys if aged 30-59 years
EXPLANATION:
• four cysts in both kidneys if aged > 60 years
Guidelines continue to recommend the use of IM diclofenac
in the acute management of renal colic
This man may need to be referred acutely to the surgeons for
pain relief and investigations to exclude obstruction. It would
not be suitable to start bendroflumethiazide in the initial
phase of the first episode
Q-214
A 62-year-old man with a diabetic nephropathy and
hypertension is reviewed. His current medication is insulin,
bendroflumethiazide, ramipril and amlodipine. On
examination blood pressure is 144/78 mmHg. Blood tests
reveal the following:
Extensive cysts are seen in an enlarged kidney
Na+ 139 mmol/l A. Take no further action
B. Send a 24-urine sample for protein estimation
K+ 4.9 mmol/l
C. Refer to nephrology
Urea 12.8 mmol/l
D. Refer to urology
Creatinine 215 µmol/l E. Confirm with urine microscopy
eGFR 29 ml/min
ANSWER:
Refer to urology
Renal function was similar to 3 months ago. What is the
most appropriate action?
EXPLANATION:
The incidence of non-visible haematuria is similar in patients
No change to his medication
taking warfarin to the general population therefore these
Switch bendroflumethiazide to furosemide
patients should be investigated as normal.
Add a beta-blocker
Add spironolactone
Most haematuria protocols suggest sending younger
Stop ramipril
patients (e.g. < 40 years) to nephrology initially.
ANSWER:
Please see Q-202 THROUGH 204 for Haematuria
Switch bendroflumethiazide to furosemide
Q-217
EXPLANATION:
A 43-year-old man presents with severe, episodic pain in his
As the eGFR is 29 ml/min switching bendroflumethiazide to
right loin region. Urine dipstick is positive for blood and you
furosemide would be the next step in controlling his blood
suspect a diagnosis of ureteric colic. At the current time he
pressure. Please see the guidelines in the external links
does not require admission. You prescribe oral naproxen and
section
arrange a non-contrast CT scan. Which one of the following
types of medication may also be beneficial in this scenario?
Please see Q-200 for Chronic Kidney Disease: Hypertension
A. Nitrate
Q-215
B. Benzodiazepine
A patient with chronic kidney disease stage 4 is started on
C. Alpha-adrenergic blocker
lisinopril. Bloods are checked two weeks later. There have
D. Beta-blocker
been no other changes to his medication and on
E. Corticosteroid
examination the patient is volume replete. According to
NICE, up to what increase in creatinine is acceptable
ANSWER:
following the introduction of an ACE inhibitor?
Alpha-adrenergic blocker
A. No increase
EXPLANATION:
B. 5%
Calcium channel blockers are also sometimes used to aid the
C. 10%
spontaneous passage of the stone.
D. 15%
E. 30%
Please see Q-201 for Renal Stones: Management
ANSWER:
Q-218
30%
A 43-year-old man has a work-up for hypertension. He has
found to have blood + on a urine dipstick of a freshly voided
EXPLANATION:
sample. Which one of the following may account for this
Please see Q-200 for Chronic Kidney Disease: Hypertension
finding?
Q-216
A. Smoking
A 62-year-old man attends your clinic. He has a history of
B. Exercise
hypertension and atrial fibrillation for which he is
C. Obesity
anticoagulated with warfarin. A urine dipstick taken 8 weeks
D. Eating red meat the previous day
ago during a routine hypertension clinic appointment
E. Use of ramipril
showed blood + with no protein or leucocytes. This result
has been repeated on two further occasions. What is the
ANSWER:
most appropriate action?
Exercise
EXPLANATION:
Please see Q-202 THROUGH 204 for Haematuria
Q-219
A 24-year-old man who has a sister with adult polycystic
kidney disease asks his GP if he could be screened for the
disease. What is the most appropriate screening test?
ANSWER: Treatment
Ultrasound abdomen • analgesia for arthralgia
• treatment of nephropathy is generally supportive. There
EXPLANATION: is inconsistent evidence for the use of steroids and
Ultrasound is the screening test for adult polycystic kidney immunosuppressants
disease
Genetic testing is still not routinely recommended for Prognosis
screening family members • usually excellent, HSP is a self-limiting condition,
especially in children without renal involvement
Please see Q-211 for ADPKD • around 1/3rd of patients have a relapse
Q-220
A 15-year-old girl presents with a palpable purpuric rash
over her lower limbs accompanied by polyarthralgia
following a recent sore throat. What is the most likely
diagnosis?
A. Rubella
B. Measles
C. Erythema multiforme
D. Idiopathic thrombocytopenic purpura
E. Henoch-Schonlein purpura
ANSWER:
Henoch-Schonlein purpura
EXPLANATION:
HENOCH-SCHONLEIN PURPURA
Henoch-Schonlein purpura (HSP) is an IgA mediated small
vessel vasculitis. There is a degree of overlap with IgA
nephropathy (Berger's disease). HSP is usually seen in children
following an infection.
Features
• palpable purpuric rash (with localized oedema) over
buttocks and extensor surfaces of arms and legs
• abdominal pain
• polyarthritis
• features of IgA nephropathy may occur e.g. haematuria,
renal failure
Q-221 A. Albumin:creatinine ratio (ACR) is more sensitive than
You are reviewing a 65-year-old for his chronic kidney protein:creatinine ratio (PCR)
disease review. He has been on haemodialysis for chronic B. An ACR of 30 mg/mmol is approximately equal to a PCR
kidney disease for the past 6 years. What is he most likely to of 50 mg/mmol
die from? C. An ACR sample is collected over 24 hours
D. Women typically have higher ACR values
A. Hyperkalaemia E. An ACR of 3.9 mg/mmol in a diabetic man is clinically
B. Malignancy significant
C. Dilated cardiomyopathy
D. Dialysis related sepsis ANSWER:
E. Ischaemic heart disease An ACR sample is collected over 24 hours
ANSWER: EXPLANATION:
Ischaemic heart disease CHRONIC KIDNEY DISEASE: PROTEINURIA
Proteinuria is an important marker of chronic kidney disease,
EXPLANATION: especially for diabetic nephropathy. NICE recommend using
CKD on haemodialysis - most likely cause of death is IHD the albumin:creatinine ratio (ACR) in preference to the
Cardiovascular events account for 50% of the mortality in protein:creatinine ratio (PCR) when identifying patients with
patients receiving dialysis. proteinuria as it has greater sensitivity. For quantification and
monitoring of proteinuria, PCR can be used as an alternative,
CHRONIC KIDNEY DISEASE: CAUSES although ACR is recommended in diabetics. Urine reagent
strips are not recommended unless they express the result as
Common causes of chronic kidney disease an ACR
• diabetic nephropathy
• chronic glomerulonephritis Approximate equivalent values
• chronic pyelonephritis
• hypertension ACR (mg/mmol) PCR (mg/mmol) Urinary protein excretion (g/24 h)
• adult polycystic kidney disease 30 50 0.5
70 100 1
Q-222
A 6-year-old boy is diagnosed as having nephrotic syndrome. Collecting an ACR sample
A presumptive diagnosis of minimal change • by collecting a 'spot' sample it avoids the need to collect
glomerulonephritis is made. What is the most appropriate urine over a 24 hour period in order to detect or quantify
treatment? proteinuria
• should be a first-pass morning urine specimen
A. Cyclophosphamide • if the initial ACR is between 3 mg/mmol and 70 mg/mmol,
B. Albumin infusion this should be confirmed by a subsequent early morning
C. Plasma exchange sample. If the initial ACR is 70 mg/mmol or more, a repeat
D. Renal biopsy followed by prednisolone sample need not be tested.
E. Prednisolone
Interpreting the ACR results
ANSWER: • the NICE guidelines state 'regard a confirmed ACR of 3
Prednisolone mg/mmol or more as clinically important proteinuria'